Sei sulla pagina 1di 73

1

A patient was referred from the oncology service because of tumoral lesions. There is no bone marrow
involvement. What is the stage on this patient?
1

IIA
2

IIB
3

IIIA
4

IIIB
5

IV
Q/Q(M)-482878 Report a Problem
A patient was referred from the oncology service because of tumoral lesions. There is no bone marrow
involvement. What is the stage on this patient?
2

IIB
This patient has the tumor stage of mycosis fungoides(MF). Patients with tumoral lesions are
automatically classified as IIB. IA is the limited patch or plaque disease with less than 10% BSA involved.
IB is the generalized patch/plaque stage with more than 10%BSA but less than 80% BSA involved. IIA
implies lymph node involvement. IIIA is erythroderma without blood involvement. IIIB is erythroderma
with low blood tumor burden. The stage IV of MF is divided in three substages: IVA1(high blood tumor
burden-Sezary syndrome), IVA2(very abnormal nodes), IVB(visceral involvement).
Q/Q(M)-482878 Report a Problem
A young woman presents with a single small, firm, umbilicated papule on the face. Biopsy reveals a well-
circumscribed lesion located in the upper dermis. Strands of basaloid cells are seen surrounded by fibrotic
or desmoplastic stroma. Horn cysts and foci of sebaceous cells and calcification are also noted. How
should this patient and her lesion be treated?
1

Reassurance and no further treatment
2

Close clinical follow-up
3

Topical 5-fluorouracil
4

Cryotherapy
5

Local surgical excision
Q/Q(M)-476105 Report a Problem

A young woman presents with a single small, firm, umbilicated papule on the face. Biopsy reveals a well-
circumscribed lesion located in the upper dermis. Strands of basaloid cells are seen surrounded by fibrotic
or desmoplastic stroma. Horn cysts and foci of sebaceous cells and calcification are also noted. How
should this patient and her lesion be treated?
5

Local surgical excision
This patient has a desmoplastic trichoepithelioma. Desmoplastic trichoepithelioma is a variant of
trichoepithelioma, an uncommon adnexal tumor with differentiation toward hair structures. It presents
commonly in young women as a small, firm, umbilicated papule on the face. Local surgical excision is the
treatment of choice.
Q/Q(M)-476105 Report a Problem

Dermoscopic features suggestive of malignant melanoma include:
1

Presence of 2 or more colors within the lesion
2

Blue-whitish veil
3

Asymmetric radial streaming
2

4

Abrupt interruption of pigment network in the periphery
5

All of these answers are correct
Q/Q(M)-474426 Report a Problem

Dermoscopic features suggestive of malignant melanoma include:
5

All of these answers are correct
Asymmetry, multicomponent pattern, blue-whitish veil, parallel-ridge pattern, atypical pigment network,
uneven radial streaming, localized irregular and diffuse pigmentation, irregularly distributed globules, and
regression structures are all dermoscopic features suggestive of malignant melanoma.
Q/Q(M)-474426 Report a Problem

All of the following are true of thick melanomas (>3 mm) except:
1

Predominantly nodular type
2

Women affected more than men
3

Predilection for the head and neck
4

Mainly in older patients (>50 years)
5

Associated with fewer nevi
Q/Q(M)-477153 Report a Problem

All of the following are true of thick melanomas (>3 mm) except:
2

Women affected more than men
According to a study performed by Chamberlain, et.al., thick melanomas (> 3 mm) were predominantly
nodular in type. They occurred in older men, mostly on the head and neck and were associated with fewer
nevi.
Q/Q(M)-477153 Report a Problem

Which syndrome is characterized by multiple keratoacanthomas beginning in childhood?
1

Muir-Torre
2

Ferguson Smith
3

Rombo
4

Grybowski
5

Keratoacanthoma centrifugum marginatum
Q/Q(M)-482675 Report a Problem

Which syndrome is characterized by multiple keratoacanthomas beginning in childhood?
2

Ferguson Smith
Ferguson Smith is familial (AD), and is characterized by regressing keratoacanthomas beginning in
childhood. Grybowski is a non-familial disorder in which patients develop generalized eruptive
keratoacanthomas beginning between age 40 to 60. Keratoacanthoma centrifugum marginatum is most
commonly a solitary lesion occurring later in life. Muir Torre is a syndrome characterized by
gastrointestinal or genitourinary neoplasms which precede development of sebaceous neoplasms and
keratoacanthomas by one to two decades. Rombo is not associated with keratoacanthomas, but is
associated with multiple basal cell carcinomas, atrophoderma vermiculatum, hypotrichosis, and milia.
Q/Q(M)-482675 Report a Problem
3

A patient presents with tender papules with a pseudo-Darier's sign. She has other family members with the
same condition. Screening should be performed to rule out which malignancy?
1

Gastric carcinoma
2

Ovarian carcinoma
3

Renal cancer
4

Testicular cancer
5

Lung cancer
Q/Q(M)-476648 Report a Problem

A patient presents with tender papules with a pseudo-Darier's sign. She has other family members with the
same condition. Screening should be performed to rule out which malignancy?
3

Renal cancer
Reed's syndrome is an autosomal dominant disease with incomplete penetrance characterized by uterine
and cutaneous leiomyomas. Clinically, leiomyomas appear as flesh colored or pinkish-brown dermal
papules or nodules that range from 0.2 to 2.0 cm in diameter. Their presentation may otherwise be quite
variable. They may be isolated or many in number, may be variably distributed or dermatomal, and may
be asymptomatic or painful in response to pressure or cold. The predisposition gene for Reeds
syndrome has been localized to chromosome 1q42.3-43 and the gene encoding fumarate hydratase.
Currently, the United States National cancer Institute is recommending screening for all patients with
leiomyomatosis to evaluate for an occult renal malignancy.
Q/Q(M)-476648 Report a Problem
Which of the following ethnic groups are commonly diagnosed with dermatosis papulosa nigra:
1

Asians
2

Hispanics
3

African-Americans & Hispanic patients
4

Caucasians
5

No difference between ethnic groups
Q/Q(M)-474394 Report a Problem

Which of the following ethnic groups are commonly diagnosed with dermatosis papulosa nigra:
3

African-Americans & Hispanic patients
Characterized by the presence of multiple, small, hyperpigmented, sessile SKs on the face, DPN is
typically diagnosed on the African-American and Hispanic population.
Q/Q(M)-474394 Report a Problem

Which of the following is an immunhistochemical marker for Merkel Cell Carcinoma?:
1

S-100
2

Vimentin
3

HMB-45
4

Neuron specific enolase
5

All of these answers are correct
Q/Q(M)-474400 Report a Problem

4

Which of the following is an immunhistochemical marker for Merkel Cell Carcinoma?:
4

Neuron specific enolase
Neuron specific enolase stains merkel cells. Vimentin stains melanocytic lesions, sarcomas and
lymphomas. S-100 and HMB-45 stains melanocytic lesions, such as melanoma.
Q/Q(M)-474400 Report a Problem

Which of the following is a common location of melanoma in women?
1

Chest
2

Lower legs
3

Genitals
4

Digits
5

Scalp
Q/Q(M)-474446 Report a Problem
Which of the following is a common location of melanoma in women?
2

Lower legs
The most common locations of MM in women are the back, lower legs and upper extremities.
Q/Q(M)-474446 Report a Problem

A patient has a malignant melanoma 1.6mm thick with ulceration and a micrometastasis in 1 node. The
patient's staging according to the American Joint Committee on Cancer Staging System is:
1

IIC
2

IIIA
3

IIIB
4

IIIC
5

IV
Q/Q(M)-476101 Report a Problem

A patient has a malignant melanoma 1.6mm thick with ulceration and a micrometastasis in 1 node. The
patient's staging according to the American Joint Committee on Cancer Staging System is:
3

IIIB
An ulcerated tumor of any size with micrometastasis in 1 node is T1-4b N1a M0. This corresponds to
stage IIIB.
Q/Q(M)-476101 Report a Problem
Compared with the general population, what is the overall risk of developing cutaneous and systemic
malignancies in organ transplant recipient?
1

4 fold
2

10 fold
3

25 fold
4

50 fold
5

100 fold
Q/Q(M)-477383 Report a Problem

5

Compared with the general population, what is the overall risk of developing cutaneous and systemic
malignancies in organ transplant recipient?
1

4 fold
Organ transplant recipients are at increased risk of having both systemic and cutaneous develop. The
estimated increased risk is 3-4 times that of the general population. However, the risk of skin cancer alone
is much higher (e.g. SCC 65x increase, BCC 10-fold increase, Melanoma 3.4x increase). This
increased risk is thought to be partially due to the immunosuppressant agents used to prevent graft
rejection.
Q/Q(M)-477383 Report a Problem

A 30 year-old gentleman presents with multiple self healing lesions similar to the one pictured. Other
family families also have the same disease. What is the most likely diagnosis?
1

Epidermodysplasia verruciformis
2

Ferguson-Smith syndrome
3

Nevoid basal cell carcinoma syndrome
4

Dyskeratosis congenital
5

Basex syndrome
Q/Q(M)-476838 Report a Problem


A 30 year-old gentleman presents with multiple self healing lesions similar to the one pictured. Other
family families also have the same disease. What is the most likely diagnosis?
2

Ferguson-Smith syndrome
Ferguson-Smith syndrome is a rare autosomal dominant condition that is characerized by multiple
keratoacanthomas found in sun-exposed areas. Lesions typically regress over weeks to months.
Q/Q(M)-476838 Report a Problem

The most common location of superficial spreading melanoma in men is:
1

Lower legs
2

Back
3

Upper extremities
4

Head and neck
5

Digits
Q/Q(M)-474444 Report a Problem

6

The most common location of superficial spreading melanoma in men is:
2

Back
Superficial spreading melanoma may be located anywhere, but the back is the most common site in male
patients.
Q/Q(M)-474444 Report a Problem

Imiquimod is an immune response modifier that stimulates innate and cell mediated immune pathways. It
induces all of the following cytokines EXCEPT:
1

IL-1
2

IL-4
3

IL-5
4

IL-6
5

IL-8
Q/Q(M)-476109 Report a Problem

Imiquimod is an immune response modifier that stimulates innate and cell mediated immune pathways. It
induces all of the following cytokines EXCEPT:
2

IL-4
Imiquimod induces the synthesis and release of cytokines such as interleukins 1, 5, 6, 8, 10, and 12,
among others.
Q/Q(M)-476109 Report a Problem

What kind of T-lymphocyte is the most common neoplastic cell in Cutaneous T-Cell Lymphoma:
1

CD4
2

CD8
3

Natural killer
4

None of these answers are correct
5

All of these answers are correct
Q/Q(M)-474420 Report a Problem

What kind of T-lymphocyte is the most common neoplastic cell in Cutaneous T-Cell Lymphoma:
1

CD4
CTCL is a neoplasm of helper T cells (CD4) that originates in the skin.
Q/Q(M)-474420 Report a Problem

The risk of metastasis from SCC increases with:
1

Tumor size
2

Depth of invasion
3

Degree of differentiation
4

Immunosupression
5

All of these answers are correct
Q/Q(M)-474419 Report a Problem
7

The risk of metastasis from SCC increases with:
5

All of these answers are correct
Tumor size (greater than 2 cms), location (lips, ears, eyelids), depth of invasion, degree of differentiation,
perineural invasion, immunosupression, as well as recurrent tumors and those that arise in areas of chronic
inflammation are all factors that increase the risk of metastasis.
Q/Q(M)-474419 Report a Problem

All of the following statements regarding Basal Cell Nevus Syndrome are true EXCEPT:
1

Associated tumors include medulloblastoma
2

It is inherited in an autosomal recessive manner
3

Hypertelorism is a feature
4

The affected gene is mutated in 30-40% of sporadic basal cell carcinomas
5

All of these answers are correct
Q/Q(M)-476111 Report a Problem

All of the following statements regarding Basal Cell Nevus Syndrome are true EXCEPT:
2

It is inherited in an autosomal recessive manner
Basal cell nevus syndrome (Gorlin Syndrome) is caused by a mutation in the PTCH gene, located on
chromosome 9q22. This gene is found to be mutated in 30-40% of sporadic basal cell carcinomas. Gorlin
Syndrome is inherited in an autosomal dominant pattern. Tumors associated with this sydrome include
medulloblastoma and meningioma. It is characterized by the appearance of multiple BCCs during
childhood, odontogenic keratocysts of the jaw, and skeletal defects (including frontoparietal bossing and
hypertelorism, among others).
Q/Q(M)-476111 Report a Problem

Which of the following features of thin melanomas (<1 mm thick) has been associated with an increased
risk of metastasis?
1

Regression
2

Location
3

Size of lesion
4

Gender
5

P53 expression
Q/Q(M)-477138 Report a Problem

Which of the following features of thin melanomas (<1 mm thick) has been associated with an increased
risk of metastasis?
1

Regression
Regression in malignant melanoma appears histologically as a focal area of fibrosis with lymphocytes and
melanophages in the papillary dermis. The presence of regression precludes accurate measurement of true
thickness of the melanoma. Extensive regression in thin melanomas has a significant association for the
risk of metastases.
Q/Q(M)-477138 Report a Problem
Pseudorosettes in Merkel cell carcinoma:
1

Are seen in the trabecular variant
8

2

Are seen in the intermediate-cell type
3

Are seen in the small-cell type
4

All of these answers are correct
5

None of these answers are correct
Q/Q(M)-476103 Report a Problem

Pseudorosettes in Merkel cell carcinoma:
1

Are seen in the trabecular variant
There are three histologic patterns of MCC: trabecular, intermediate-cell type, and small-cell type. The
trabecular variant consists of interconnecting trabeculae separated by strands of connective tissue.
Pseudorosettes may be seen in this type.
Q/Q(M)-476103 Report a Problem

UVB induced mutations on the PTCH gene is associated with the development of:
1

BCC
2

Merkel cell carcinoma
3

Angiosarcoma
4

BCC and Merkel cell carcinoma
5

Merkel cell carcinoma and Angiosarcoma
Q/Q(M)-474402 Report a Problem

UVB induced mutations on the PTCH gene is associated with the development of:
1

BCC
The p53 and PTCH genes are the major targets of UVB for the development of BCC. Other genes
involved include the Smoothened-activating mutations, and PTCH2 mutations.
Q/Q(M)-474402 Report a Problem

Which of the following is the most common initial site of metastasis from a primary BCC?
1

Lungs
2

Regional lymph nodes
3

Bone
4

Liver
5

Pleura
Q/Q(M)-474439 Report a Problem

Which of the following is the most common initial site of metastasis from a primary BCC?
2

Regional lymph nodes
The metastatic potential of BCC is very low with rates ranging from 0.0028 to 0.1%. The head and neck
region is the most frequent location of the primary tumor with regional lymph nodes being the most
common site of metastasis. The lungs, bone, liver, and pleural are also potential sites of metastasis.
Q/Q(M)-474439 Report a Problem

9

What temperature must be achieved for adequate treatment of a small superficial squamous cell carcinoma
with cryotherapy?
1

-10 degrees Celcius
2

-20 degrees Celcius
3

-30 degrees Celcius
4

-40 degrees Celcius
5

-50 degrees Celcius
Q/Q(M)-479616 Report a Problem

What temperature must be achieved for adequate treatment of a small superficial squamous cell carcinoma
with cryotherapy?
5

-50 degrees Celcius
Cryosurgery destroys tumor if it is frozen to -40 to -70 degrees Celcius, for at least 2 cycles with a 60
second thaw. Benign lesions can be destroyed at temperatures around -25 degrees Celcius.
Q/Q(M)-479616 Report a Problem
The most common locations of microcystic adnexal carcinoma include all of the following, except:
1

Perioral
2

Nasolabial
3

Trunk
4

Periorbital
5

Perioral,nasolabial, and periorbital
Q/Q(M)-474987 Report a Problem

The most common locations of microcystic adnexal carcinoma include all of the following, except:
3

Trunk
Microcystic adnexal carcinomas are more commonly located in the perioral, nasolabial, or periorbital
areas.
Q/Q(M)-474987 Report a Problem

Which of the following are features of microcystic adnexal carcinoma that help distinguish it from
desmoplastic trichoepitheliomas?
1

Deep subcutaneous infiltration
2

Perineural invasion
3

CEA positive staining
4

Commonly located on the face
5

Deep subcutaneous infiltration,perineural invasion, and CEA positive staining
Q/Q(M)-474988 Report a Problem

Which of the following are features of microcystic adnexal carcinoma that help distinguish it from
desmoplastic trichoepitheliomas?
5

Deep subcutaneous infiltration,perineural invasion, and CEA positive staining
Desmoplastic trichoepitheliomas is one of the differential diagnosis for MAC. MAC show deep
subcutaneous and perineural invasion, as well as CEA positive staining, all features that may help
10

differentiate it from desmoplastic trichoepitheliomas. Both, MAC and desmoplastic trichoepitheliomas are
commonly located on the face.
Q/Q(M)-474988 Report a Problem

The following lesion is the classic presentation of:
1

BCC
2

Merkel cell carcinoma
3

Melanoma
4

CTCL
5

Keratoacanthoma
Q/Q(M)-474902 Report a Problem


The following lesion is the classic presentation of:
5

Keratoacanthoma
Keratoacanthomas present as a solitary, firm, dome-shaped papule with a cratiform center.
Q/Q(M)-474902 Report a Problem

Which one of the following malignancies is associated with HPV infection?
1

Verrucous carcinoma
2

Metastatic melanoma
3

Basal cell carcinoma
4

Sebaceous carcinoma
5

Atypical fibroxanthoma
Q/Q(M)-477339 Report a Problem

Which one of the following malignancies is associated with HPV infection?
1

Verrucous carcinoma
Verrucous carcinomas are low-grade carcinomas which are slow-growing and metastasize very late in the
course. The presence of HPV has been demonstrated in cases both by electron microscopy and DNA
hybridization.
Q/Q(M)-477339 Report a Problem

In which of the following ethnic groups is squamous cell carcinoma the most common type of skin
cancer?
1

Asian Indians
11

2

Caucasians
3

Hispanics
4

Japanese
5

Chinese
Q/Q(M)-482498 Report a Problem

In which of the following ethnic groups is squamous cell carcinoma the most common type of skin
cancer?
1

Asian Indians
SCC is the most common type of skin cancer in Asian Indians and Blacks. BCC is the most common type
of skin cancer in Caucasians, Japanese, Chinese, and Hispanic people.
Q/Q(M)-482498 Report a Problem

Which of the following immunohistochemistry marker is negative in angiosarcomas?
1

CD31
2

CEA
3

Cytokeratin
4

CD34
5

Factor VIII
Q/Q(M)-474424 Report a Problem
Which of the following immunohistochemistry marker is negative in angiosarcomas?
2

CEA
Angiosarcomas are CD31, CD34, factor VIII, and cytokeratin positive. The carcinoembryonic antigen
(CEA) stains positive in Pagets disease, metastatic adenocarcinoma, and tumors with eccrine
differentiation.
Q/Q(M)-474424 Report a Problem
What is the most common site of metastasis for this dermal tumor?
1

Lung
2

Brain
3

Kidney
4

Liver
5

Bone
Q/Q(M)-476843 Report a Problem

What is the most common site of metastasis for this dermal tumor?
12

1

Lung
Dermatofibrosarcoma protuberans is a rare, low-grade dermal sarcoma. Typically, the lesion occurs as a
painless subcutaneous mass that grows slowly. This malignancy typically has lateral spread but invade
deep. Metastasis is rare but has been reported to the lung.
Q/Q(M)-476843 Report a Problem

All of the following cytokines have demonstrated therapeutic benefit in the treatment of melanoma
EXCEPT:
1

IFN-alpha
2

IL-2
3

TNF-alpha
4

IL-10
5

GM-CSF
Q/Q(M)-477183 Report a Problem

All of the following cytokines have demonstrated therapeutic benefit in the treatment of melanoma
EXCEPT:
5

GM-CSF
IFN-alpha, IL-2, TNF-alpha, and IL-10 have all been demonstrated to have some therapeutic benefit in the
treatment of melanoma. IL-10 has been used to treat inflammatory disorders such as atopic dermatitis and
psoriasis.
Q/Q(M)-477183 Report a Problem

Which one of the following agents has demonstrated potential benefit as a chemopreventive to UV-
induced skin cancer?
1

Prostaglandin E2
2

Vitamin D
3

Arachidonic acid
4

Celecoxib
5

Vitamin E
Q/Q(M)-477331 Report a Problem

Which one of the following agents has demonstrated potential benefit as a chemopreventive to UV-
induced skin cancer?
4

Celecoxib
Cyclooxygenase-1 and -2 and enzymes that catalyze the conversion of arachidonic acid to prostaglandins.
It is believed the prostaglandin E2 (PGE2), whose levels are increased by ultraviolet irradiation, is pro-
inflammatory and may contribute to skin carcinogenesis. In a study by Orengo et.al., hairless mice who
were given celecoxib were found to have a significantly longer latency period between exposure to
ultraviolet light and the development of skin carcinomas.
Q/Q(M)-477331 Report a Problem

Intermittent sun exposure with painful sunburns is a predisposing factor for the development of:
1

Atypical nevi
13

2

Seborrheic keratosis
3

Malignant melanoma
4

Atypical nevi and Malignant melanoma
5

All of these answers are correct
Q/Q(M)-474429 Report a Problem

Intermittent sun exposure with painful sunburns is a predisposing factor for the development of:
4

Atypical nevi and Malignant melanoma
It has been reported that the risk for the development of atypical nevi and melanoma is higher than
twofold with a history of five or more episodes of painful sunburn during adolescence.
Q/Q(M)-474429 Report a Problem

The incidence of cutaneous squamous cell carcinoma in organ transplant recipients is increased by how
much compared with the general population?
1

2 fold
2

5 fold
3

10 fold
4

20 fold
5

65 fold
Q/Q(M)-477368 Report a Problem
The incidence of cutaneous squamous cell carcinoma in organ transplant recipients is increased by how
much compared with the general population?
5

65 fold
The risk of skin cancer in organ transplant patients is dramatically increase and may be more aggressive.
In one study, SCCs had a 65x increased incidence, BCC 10x, and melanoma 3.4x.
Q/Q(M)-477368 Report a Problem
Which of the following neoplasms has demonstrated an association with HTLV-1 infection?
1

Mycosis fungoides
2

Adult T cell lymphoma
3

Follicular lymphoma
4

Multiple myeloma
5

Hodgkins disease
Q/Q(M)-477137 Report a Problem

Which of the following neoplasms has demonstrated an association with HTLV-1 infection?
2

Adult T cell lymphoma
Mycosis fungoides is a rare form of cutaneous T-cell lymphoma, the etiology of which is not completely
known. Adult T Cell Lymphoma is also a type of T cell neoplasm that has been linked to HTLV-1. Adult
T-cell lymphoma may have an acute and chronic, smoldering form. The chronic, smoldering form can be
difficult to distinguish from mycosis fungoides.
Q/Q(M)-477137 Report a Problem

14

All of the following statements are true regarding angiosarcomas EXCEPT:
1

They occur more commonly in Caucasians than in non-Caucasians
2

Men are more often affected than women
3

They are rarely symptomatic
4

Radiation is usually employed after surgical excision
5

Cervical lymph nodes are a common site of metastases
Q/Q(M)-476124 Report a Problem

All of the following statements are true regarding angiosarcomas EXCEPT:
3

They are rarely symptomatic
Angiosarcomas are very rare, aggressive vascular tumors. They occur most commonly in the head and
neck region of white, elderly individuals. Men are more commonly diagnosed with this neoplasm. The
lesion initially arises as a painless, purple macule or plaque on the scalp or face. Later on it becomes an
elevated bluish or purple nodule that may ulcerate. Common symptoms include bleeding, edema, and
ultimately pain. Cervical lymph node and hematogenous metastases commonly occur. Wide surgical
excision is the treatment of choice, with radiation therapy usually employed after surgical excision.
Q/Q(M)-476124 Report a Problem

Which neoplasm is associated with the Stewart-Treves syndrome?
1

Renal leiomyomas
2

Basal cell carcinoma
3

Angiosarcoma
4

Keratoacanthoma
5

T cell lymphoma
Q/Q(M)-477129 Report a Problem

Which neoplasm is associated with the Stewart-Treves syndrome?
3

Angiosarcoma
Stewart-Treves syndrome is the development of angiosarcoma in the setting of chronic lymphedema.
Originally, named after radical mastectomy for the treatment of breast cancer. The term applies to the
development of angiosarcoma in any chronic lymphedematous condition.
Q/Q(M)-477129 Report a Problem
Which test should be used to detect monoclonal gene rearrangements in cutaneous T-cell lymphoma?
1

Northern blot
2

Southern blot
3

Western blot
4

ELISA
5

Electrophoresis
Q/Q(M)-476531 Report a Problem

Which test should be used to detect monoclonal gene rearrangements in cutaneous T-cell lymphoma?
2

Southern blot
Monoclonality and gene rearrangements can be detected with a Southern blot. Southern blots can be used
15

to detect specific DNA fragments by gel-transfer hybridization.
Q/Q(M)-476531 Report a Problem

The treatment of choice for this lesion shown is:
1

Radiation therapy
2

Imiquimod
3

5 Flourouracil
4

Wide excision with 2cm margins
5

Mohs surgery
Q/Q(M)-478126 Report a Problem


The treatment of choice for this lesion shown is:
5

Mohs surgery
The treatment of choice for Dermaotfibrosarcoma protuberans is Mohs surgery. Radiation therapy has
been used, however has limited value as solitary therapy for thsi tumor. Raditationtherapy cna be used as
an adjunct to wide surgical excision. Calssically these tumors should be excised wtih 3cm margins. The
recurrence rate associated with these tumors can be 10-20 percent with wide excision with 3 cm margins.
With Mohs surgery, the recurrence rate ranges from 0% to 6%. 5FU and Imiquimod are not effective
modalitites in treating DFSPs, as it infiltrates deep into the subcutaneous tissue.
Q/Q(M)-478126 Report a Problem
Which of the following would you not expect to see under dermoscopy?
1

Maple leaf pattern
2

Arborizing blood vessels
3

Blue-grey ovoid nests
4

Orange crust
5

Milky red globules
Q/Q(M)-476837 Report a Problem

Which of the following would you not expect to see under dermoscopy?
16

5

Milky red globules
Dermoscopy is a useful tool in differentiating a pigmented basal cell carcinoma from melanoma. Basal
cell carcinomas may have arborizing blood vessels, maple leaf pattern, blue-grey ovoid nests, and orange
crust or ulcer. Milky red globules are sometimes seen in melanoma.
Q/Q(M)-476837 Report a Problem

A 56-year old woman with a history significant for chronic lymphedema after radical mastectomy twelve
years ago presents with this growth on her arm. What is the diagnosis?
1

Angiosarcoma
2

Bacillary angiomatosis
3

Castleman's syndrome
4

Kaposi's sarcoma
5

Metastatic breast carcinoma
Q/Q(M)-476834 Report a Problem

A 56-year old woman with a history significant for chronic lymphedema after radical mastectomy twelve
years ago presents with this growth on her arm. What is the diagnosis?
1

Angiosarcoma
Angiosarcoma may occur in association with chronic lymphedematous states. Stuart-Treves syndrome
applies to the specific condition of an angiosarcoma arising on the upper arm after radical mastectomy.
Q/Q(M)-476834 Report a Problem

A 60 year-old female presents with a well-demarcated, scaly, erythematous plaque on her right shin. The
biopsy shows full thickness epidermal atypia with scattered mitotic figures and overlying parakeratosis.
Howerver, the basement membrane remains intact. According to the aforementioned information, the
diagnosis is:
1

BCC
2

SCC
3

Bowen's disease
4

Angiosarcoma
5

CTCL
17

Q/Q(M)-474903 Report a Problem
A 60 year-old female presents with a well-demarcated, scaly, erythematous plaque on her right shin. The
biopsy shows full thickness epidermal atypia with scattered mitotic figures and overlying parakeratosis.
Howerver, the basement membrane remains intact. According to the aforementioned information, the
diagnosis is:
3

Bowen's disease
Bowens disease arising on the lower limbs is frequently found in women; whereas lesions located on
the ears and scalp are more common in men. The epidermal dysplasia does not interrupt the basement
membrane confirming the diagnosis of SCC in situ.
Q/Q(M)-474903 Report a Problem

All of the followings can be used for treatemt of this condition except
1

Cryotherapy
2

Topical Imiquimod 5% cream
3

Topical 5-flurouracil
4

Topical retinoids
5

Surgical excision
Q/Q(M)-482126 Report a Problem

All of the followings can be used for treatemt of this condition except
5

Surgical excision
Attached picture is disseminated superficial actinic porokeratosis (DSAP) which is the most common type
of all porokeratosis, with multiple thin papules appearing most commonly on the legs of adult women.
Many treatments have been used for this condition which include: cryotherapy, topical 5-fluorouracil (5-
FU), topical retinoids, CO2 laser, and dermabrasion. Although surgical excision might be used for
treatment of other forms of porokeratosis, it is not advised in this case because of number of lesions and
increase risk of scarring. Other forms of porokeratosis are: porokeratosis of Mibelli, punctate
porokeratosis, linear, and Porokeratosis palmaris et plantaris disseminata.
Q/Q(M)-482126 Report a Problem

If left untreated, which of the following is not at risk for malignant transformation?
1

Bowenoid papulosis
2

Cutaneous horn
3

Actinic cheilitis
4

Leukoplakia
5

Stucco keratosis
Q/Q(M)-474397 Report a Problem
18


If left untreated, which of the following is not at risk for malignant transformation?
5

Stucco keratosis
Cutaneous horn is defined as a hypertrophic AK that presents as a conical protuberance arising from an
erythematous base. Actinic cheilitis results from the confluence of multiple AKs on the lips. Leukoplakia
is a clinical diagnosis and is defined as a white patch in the oral cavity. It is the most common
premalignant condition of the oral cavity. Bowenoid papulosis manifests clinially as multiple red-brown
warty papules that histologically represent high grade squamous intraepithelial lesions.
Q/Q(M)-474397 Report a Problem
Which of the following variants of mycosis fungoides is best diagnosed using a punch biopsy instead of a
broad superficial shave biopsy?
1

Woringer-Kolopp pagetoid reticulosis
2

Syringotropic mycosis fungoides
3

Ketron-Goodman pagetoid reticulosis
4

Poikilodermatous mycosis fungoides
5

Sezary syndrome
Q/Q(M)-482500 Report a Problem

Which of the following variants of mycosis fungoides is best diagnosed using a punch biopsy instead of a
broad superficial shave biopsy?
2

Syringotropic mycosis fungoides
Syringotropic and folliculotropic mycosis fungoides (MF) are the variants of MF that should be diagnosed
by punch biopsy. Both variants of pagetoid reticulosis, Woringer-Kolopp disease and the disseminated
Ketron-Goodman disease, are best diagnosed with a broad shave biopsy. Classic and poikilodermatous
MF should also be diagnosed with a shave biopsy. To rule out Sezary syndrome, flow cytometry should be
performed.
Q/Q(M)-482500 Report a Problem

Merkel cell carcinoma stains positively for:
1

Leukocyte common antigen
2

Neuron specific enolase
3

Vimentin
4

S-100
5

None of these answers are correct
Q/Q(M)-474436 Report a Problem

Merkel cell carcinoma stains positively for:
2

Neuron specific enolase
Merkel cell carcinoma is frequently diagnosed in individuals older than 50 years of age. However, there
have been reports of MCC in children and young adults.
Q/Q(M)-474436 Report a Problem

19

A 3-year-old girl presents with multiple small, angulated, firm nodules. There is a positive "tent sign" and
biopsy reveals ghost cells and germinative cells. Of the following conditions associated with multiple
pilomatricomas, which is thought to be most closely linked?
1

Turner's
2

Myotonic dystrophy
3

Rubenstein-Taybi
4

Sarcoidosis
5

Gardner's syndrome
Q/Q(M)-482439 Report a Problem

A 3-year-old girl presents with multiple small, angulated, firm nodules. There is a positive "tent sign" and
biopsy reveals ghost cells and germinative cells. Of the following conditions associated with multiple
pilomatricomas, which is thought to be most closely linked?
2

Myotonic dystrophy
Pilomatricomas are the most common superficial pediatric tumor. The occurrence of multiple
pilomatricomas has been most closely associated with the development of myotonic dystrophy(Steinert
Disease). The onset of myotonic dystrophy, an autosomal dominant disorder with variable penetrance, can
occur before or after the onset of lesions. Cigliano et al. reported myotonic dystrophy in 1 out of 2 patients
with multiple pilomatricomas; Julian et al. reported myotonic dystophy in 1 out of 4 patients. The other
syndromes associated with multiple pilomatricomas include Rubenstein-Taybi, Turner syndrome, Gardner
syndrome, and sarcoidosis.
Q/Q(M)-482439 Report a Problem

Which of the following immunohistochemical markers may be indicative of metastatic potential for
merkel cell carcinoma?
1

Chromogranin
2

Synaptophysin
3

CD44
4

CK20
5

TTF-1
Q/Q(M)-482512 Report a Problem

Which of the following immunohistochemical markers may be indicative of metastatic potential for
merkel cell carcinoma?
3

CD44
Chromogranin and synaptophysin are positive in merkel cell carcinoma along with neuron-specific
enolase and EMA. CK20 is also positive in merkel cell carcinoma and exhibits a paranuclear dot staining
pattern. According to a 1996 study, CD44 may indicate metastatic potential. TTF-1 is negative in merkel
cell carcinoma, which helps distinguish it from small cell lung carcinoma.
Q/Q(M)-482512 Report a Problem

All of the following disorders may manifest as exfoliative dermatitis except
1

Behchets disease
2

Psoriasis
3

Pemphigus foliaceus
20

4

Drug reaction
5

Sezary syndrome
Q/Q(M)-481908 Report a Problem


All of the following disorders may manifest as exfoliative dermatitis except
1

Behchets disease
The image shows erythrodermic patient with generalized desquamation of skin. Skin biopsy and blood
tests the diagnosis of Sezary syndrome. Sezary syndrome is a leukemic variant of mycosis fungoides, is
characterized by the triad of pruritic erythroderma, generalized lymphadenopathy, and the presence of
Sezary cells more than 1,000 cell/mm3 (abnormal, large hyperconvoluted lymphocytes) in peripheral
blood . Other manifestations include scaling and fissuring of palms and soles, alopecia, pruritus,
peripheral edema, and nail dystrophy. Exfoliative dermatits can be manifested in many conditions other
than Sezary syndrome inculude pemphigus foliaceus, psoriasis and drug reaction.
Q/Q(M)-481908 Report a Problem

The most common site for intra oral melanoma is?
1

buccal
2

soft palate
3

hard palate
4

gingiva
5

uvula
Q/Q(M)-482205 Report a Problem

The most common site for intra oral melanoma is?
3

hard palate
Multiple case series have shown that the hard palate (and specifically the anterior hard palate/alveolar
arch) is the highest risk location for intra-oral melanoma. Prognosis is generally worse for oral melanoma,
which is usually due to delay in diagnosis and presentation with more invasive disease.
Q/Q(M)-482205 Report a Problem

A 40 year-old female patient presents with the following lesion (see image). The biopsy report shows duct
like structures, tadpole structures within a fibrotic stroma. According to the aforementioned information,
the most likely diagnosis is:
1

Microcystic adnexal carcinoma
2

Seborrheic keratosis
21

3

Dermatofibrosarcoma protuberans
4

Merkel cell carcinoma
5

None of these answers are correct
Q/Q(M)-474991 Report a Problem

A 40 year-old female patient presents with the following lesion (see image). The biopsy report shows duct
like structures, tadpole structures within a fibrotic stroma. According to the aforementioned information,
the most likely diagnosis is:
1

Microcystic adnexal carcinoma
One of the most common locations for MAC includes the perioral area. Histologically it presents with
poorly demarcated tumor cells invading the dermal and subcutaneous tissue. Islands of basaloid
keratinocytes, horn cysts and duct structures are also seen within a desmoplastic stroma.
Q/Q(M)-474991 Report a Problem

Sezary syndrome:
1

Has characteristic Sezary cells in peripheral blood
2

Is the leukemic variant of Mycosis fungoides
3

Is characterized by the triad of pruritic erythroderma, generalized lymphadenopathy, and presence
of Sezary cells in peripheral blood
4

Is associated with a poor prognosis
5

All of these answers are correct
Q/Q(M)-474438 Report a Problem

Sezary syndrome:
5

All of these answers are correct
Sezary syndrome is the leukemic variant of CTCL characterized by the triad of pruritic erythroderma,
generalized lymphadenopathy, and presence of Sezary cells in peripheral blood. Sezary cells are
abnormal, large hyperconvoluted lymphocytes. The five year survival is estimated to be between 10-20%.
Q/Q(M)-474438 Report a Problem
Amplification of which of the following genes is associated with Merkel cell carcinoma?
1

L-Myc
2

C-Myc
3

GLI1
4

CDKN2A
5

PTEN
22

Q/Q(M)-482403 Report a Problem
Amplification of which of the following genes is associated with Merkel cell carcinoma?
1

L-Myc
The L-Myc gene has been found to be amplified in Merkel cell carcinoma but not in normal skin. C-Myc
has been found to be amplified in neuroblastoma. GLI1 is a transcription factor involved in hedgehog
signaling and a potential target in basal cell carcinomas. CDKN2A is implicated in familial forms of
melanoma that are associated with pancreatic cancer. PTEN is a tumor suppressor gene that is mutated in
Cowden Disease, Bannayan-Riley-Ruvalcaba Syndrome, and Proteus Syndrome.
Q/Q(M)-482403 Report a Problem
All of the following are risk factors for metastasis from a primary squamous cell carcinoma EXCEPT:
1

Increasing tumor size
2

Location on the ear
3

Recurrent tumor
4

Tumor within ulcer
5

All of these answers are correct
Q/Q(M)-476123 Report a Problem

All of the following are risk factors for metastasis from a primary squamous cell carcinoma EXCEPT:
5

All of these answers are correct
The risk of metastasis from a primary SCC increases with tumor size (>2cm), location (lips, ear, and
eyelid, among others), depth of invasion, immunosuppression, degree of differentiation, and perineural
invasion. Recurrent tumors and SCCs arising in areas of chronic inflammation (such as osteomyelitis,
burn scars, or ulcers) are also considered high risk for metastases.
Q/Q(M)-476123 Report a Problem
What is the most likely diagnosis?
1

Acquired digital fibrokeratoma
2

Supernumery digit
3

Verruca
4

Infantile digital fibroma
5

Acrochordon
Q/Q(M)-476808 Report a Problem


23

What is the most likely diagnosis?
1

Acquired digital fibrokeratoma
The condition shown is an acquired digital fibrokeratoma. It is a pedunculated, acral lesion with collarette
which is defined by the lack of nernve twigs and bone on pathologic exam.
Q/Q(M)-476808 Report a Problem
Which phase of the cell cycle does p53 regulate?
1

G1
2

G2
3

S phase
4

Mitosis
5

Meiosis
Q/Q(M)-476595 Report a Problem

Which phase of the cell cycle does p53 regulate?
1

G1
p53 is a tumor suppressor gene which arrests cell cycle and downregulates BCL-2. Mutations in p53 are
associated with Li-Fraumeni syndrome as well as the development of squamous cell carcinomas.
Q/Q(M)-476595 Report a Problem
Which area of the face receives the most cumulative exposure to UV radiation?
1

Dorsum of nose
2

Orbital region
3

Base of the nose
4

Chin
5

Central cheek
Q/Q(M)-477225 Report a Problem
Which area of the face receives the most cumulative exposure to UV radiation?
3

Base of the nose
The most frequent sites of basal cell carcinomas in one study was the base of the nose (bordering the nasolabial
fold and extending midway up the side of the nose), orbital region, apex of the nose, ear, forehead, temporal
region, nasolabial area, and the buccal region (in order). The nose was by far the most frequent site of BCCs.
Q/Q(M)-477225 Report a Problem
This tumor, also called a Shagreen patch, is characteristic of which of the following genodermatoses?
1

Neurofibromatosis-1
24

2

Neurofibromatosis-2
3

Tuberous sclerosis
4

NAME syndrome
5

Buschke-Ollendorf
Q/Q(M)-476842 Report a Problem

This tumor, also called a Shagreen patch, is characteristic of which of the following genodermatoses?
3

Tuberous sclerosis
Tuberous sclerosis a rare genodermatosis characterized by adenoma sebaceum, seizures and mental retardation.
Other cutaneous features include hypopigmented macules, periungual fibromas, fibrous plaque of the face, cafe-
au-lait macules and connective tissue nevus.
Q/Q(M)-476842 Report a Problem
This lesion is associated with which of the following:
1

HPV 5
2

HPV 11
3

HPV 4
4

HPV 7
5

HPV 1
Q/Q(M)-477683 Report a Problem
25


This lesion is associated with which of the following:
2

HPV 11
HPV 6 and 11 are most often associated with anogenital warts or condyloma acuminata. They can present as
sessile lesions on the skin or peducnulated cauliflower lesions. HPV type 5 is associated with epidermodysplasia
verruciformis, HPV 4 is assoicatd with common warts, HPV type 7 is associated with Butcher warts, and HPV type 1
is associated with common warts.
Q/Q(M)-477683 Report a Problem
All of the following statements are true regarding angiosarcomas EXCEPT:
1

They occur more commonly in Caucasians than in non-Caucasians
2

Men are more often affected than women
3

They are rarely symptomatic
4

Radiation is usually employed after surgical excision
5

Cervical lymph nodes are a common site of metastases
Q/Q(M)-476124 Report a Problem
All of the following statements are true regarding angiosarcomas EXCEPT:
3

They are rarely symptomatic
Angiosarcomas are very rare, aggressive vascular tumors. They occur most commonly in the head and neck region
of white, elderly individuals. Men are more commonly diagnosed with this neoplasm. The lesion initially arises as a
painless, purple macule or plaque on the scalp or face. Later on it becomes an elevated bluish or purple nodule
that may ulcerate. Common symptoms include bleeding, edema, and ultimately pain. Cervical lymph node and
hematogenous metastases commonly occur. Wide surgical excision is the treatment of choice, with radiation
therapy usually employed after surgical excision.
Q/Q(M)-476124 Report a Problem
When performing a biopsy of a suspected keratoacanthoma:
26

1

Fusiform incision through the entire KA may be performed
2

It is necessary to biopsy down to subcutaneous fat
3

A full-thickness shave biopsy is acceptable
4

A complete excisional biopsy may be performed
5

All of these answers are correct
Q/Q(M)-476099 Report a Problem
When performing a biopsy of a suspected keratoacanthoma:
5

All of these answers are correct
When considering a keratoacanthoma, it is important to obtain a biopsy of the specimen down to the
subcutaneous fat. This can be achieved either by complete excisional biopsy, full-thickness shave biopsy, or
fusiform incision through the entire KA including its center and sides.
Q/Q(M)-476099 Report a Problem
Currently, the surgical margin for melanomas that measure less than 2 mm in thickness is:
1

1 mm
2

0.5 cm
3

1 cm
4

2 cms
5

3 cms
Q/Q(M)-474422 Report a Problem
Currently, the surgical margin for melanomas that measure less than 2 mm in thickness is:
3

1 cm
The current surgical margins are 0.5 cms for melanoma in situ, 1 cm for melanomas that measure =2mm in
thickness.
Q/Q(M)-474422 Report a Problem
Which of the following melanoma scenarios have the best prognosis?
1

Twenty-one year old female with primary lesion located on the right lower leg
27

2

Twenty-one year old male with primary lesion located on the chest
3

Thirty-six year old male with primary lesion located on the back
4

Twenty-one year old female with primary, ulcerated lesion located on the right lower leg
5

Thirty-six year old male with primary lesion located on his left lower leg with palpable inguinal lymph nodes
Q/Q(M)-474423 Report a Problem
Which of the following melanoma scenarios have the best prognosis?
1

Twenty-one year old female with primary lesion located on the right lower leg
Increasing age and male gender have a negative effect on survival. Primary lesions located on the extremities have
a better prognosis than those located on the trunk, head or neck region. Ulceration is also considered a poor
prognostic factor. Furthermore, there is a significantly lower survival for those patients with palpable metastatic
nodes (macrometastasis) when compared to those with micrometastatic nodes (nonpalpable).
Q/Q(M)-474423 Report a Problem

Desmoplastic trichoepithelioma:
1

Is most common in middle-aged patients
2

More commonly appears in males than in females
3

Does not display foci of calcification or ossification
4

Presents as a well-circumscribed lesions located in the upper dermis
5

None of these answers are correct
Q/Q(M)-476104 Report a Problem
Desmoplastic trichoepithelioma:
4

Presents as a well-circumscribed lesions located in the upper dermis
Desmoplastic trichoepithelioma is a variant of trichoepithelioma, an uncommon adnexal tumor with
differentiation toward hair structures. It is three times more common in females and usually occurs in young
adults. The tumor is a well-circumscribed nodule lesions located in the upper dermis. Strands or columns of
basaloid cells are seen surrounded by fibrotic or desmoplastic stroma. Horn cysts may also be seen, as well as foci
of sebaceous cells, calcification and ossification.
Q/Q(M)-476104 Report a Problem
28

Which of the following immunologic drugs has been shown to increase survival of patients with stage III and stage
IV melanoma?
1

Etanercept
2

Adalimumab
3

Ustekinumab
4

Ipilimumab
5

Infliximab
Q/Q(M)-482376 Report a Problem
Which of the following immunologic drugs has been shown to increase survival of patients with stage III and stage
IV melanoma?
4

Ipilimumab
A recent study in the New England Journal of Medicine reported an increase in survival of patients with stage III
and stage IV melanoma with a new immunologic therapy agent named ipilimumab. The survival was increased
from 6 months to 10 months (P < 0.001). All the other medicines are not used to treat melanoma; they are used to
treat psoriasis, among other disease.
Q/Q(M)-482376 Report a Problem
What would be the characteristic histopathologic findings of this lesion?
1

Full thickness keratinocytic atypia
2

Cornoid lamella
3

Pale staining cells
4

Horn pseudocysts
5

Wedge shaped granular layer with lichenoid infiltrate
Q/Q(M)-476650 Report a Problem
29


What would be the characteristic histopathologic findings of this lesion?
2

Cornoid lamella
Five clinical variants of porokeratosis are recognized. They are the classic porokeratosis of Mibelli, disseminated
superficial actinic porokeratosis, prokeratosis palmaris et plantaris disseminata, linear porokeratosis, and punctate
porokeratosis. On histopathology they have varying degrees of a cornoid lamella. This appears as a column of
parakeratosis.
Q/Q(M)-476650 Report a Problem
Mohs micrographic surgery the treatment of choice for all of the following, except:
1

1 cm SCC located on the chest
2

2 cms BCC on lower extremities
3

1 cm BCC on the eyelid
4

Morpheaform BCC on the cheek
5

Recurrent BCC on the chest
Q/Q(M)-474437 Report a Problem
Mohs micrographic surgery the treatment of choice for all of the following, except:
1

1 cm SCC located on the chest
SCCs that measure 1cm on the chest are not an indication for Mohs micrographic surgery. Lesions must measure
2cms on the chest and extremities, or ?1cm for those located on the face to be considered for Mohs surgery. Due
to the aggressive growth patter and subclinical spread of morpheaform and recurrent BCCs are always and
indication for Mohs surgery.
30

Q/Q(M)-474437 Report a Problem
Periungual Squamous cell carcinoma has been linked to which HPV type(s)?
1

6, 11
2

2, 4
3

16
4

13
5

8
Q/Q(M)-477402 Report a Problem
Periungual Squamous cell carcinoma has been linked to which HPV type(s)?
3

16
Infections associated with Human Papilloma Virus can produce growths on the epithelial or mucosal surfaces.
There are over 100 strains of these viruses, and some of these strains can predispose to intraepithelial carcinomas,
particularly when involving the anal or genital mucosa. In general, HPV strains 16 and 18 are classified as his risk
virus types and can be associated with cervical cancer , oral cancer, anal cancer and periungual cancers. HPV 6 and
11 are associated with condyloma acuminata; HPV types 2, 4 are associated with common warts; HPV type 13 has
been associated with Heck's disease and HPV type 8 has been associated with epidermal dysplasia verruciformis.
Q/Q(M)-477402 Report a Problem
What cancer in women most commonly metastasizes to the skin?
1

Breast cancer
2

Medullary thyroid carcinoma
3

Glioblastoma multiforme
4

Colon adenocarcinoma
5

Cervical cancer
Q/Q(M)-476833 Report a Problem

What cancer in women most commonly metastasizes to the skin?
1

Breast cancer
31

Breast cancer is the most common cancer to metastasize to the skin in women.
Q/Q(M)-476833 Report a Problem
Dermatofibrosarcoma protuberans is:
1

Cytogenetically characterized by reciprocal translocation t(17;22)(q22;q13)
2

Factor XIIIa positive
3

CEA positive
4

CD 34 positive
5

Cytogenetically characterized by reciprocal translocation t(17;22)(q22;q13) and CD 34 positive
Q/Q(M)-474986 Report a Problem
Dermatofibrosarcoma protuberans is:
5

Cytogenetically characterized by reciprocal translocation t(17;22)(q22;q13) and CD 34 positive
A chromosomal reciprocal translocation t(17;22)(q22;q13), and supernumerary ring chromosome have been
reported as cytogenetic characteristics of DFSP. Typically DFSP is CD34 positive and factor XIIIa negative, allowing
its differentiation from dermatofibroma.
Q/Q(M)-474986 Report a Problem
What temperature must be achieved for adequate treatment of a small superficial squamous cell carcinoma with
cryotherapy?
1

-10 degrees Celcius
2

-20 degrees Celcius
3

-30 degrees Celcius
4

-40 degrees Celcius
5

-50 degrees Celcius
Q/Q(M)-479616 Report a Problem
What temperature must be achieved for adequate treatment of a small superficial squamous cell carcinoma with
cryotherapy?
5

-50 degrees Celcius
Cryosurgery destroys tumor if it is frozen to -40 to -70 degrees Celcius, for at least 2 cycles with a 60 second thaw.
32

Benign lesions can be destroyed at temperatures around -25 degrees Celcius.
Q/Q(M)-479616 Report a Problem
Which substance does p53 normally activate to promote apoptosis via inhibition of bcl-2?
1

p21
2

p16
3

Puma
4

Mdm2
5

Akt
Q/Q(M)-482363 Report a Problem
Which substance does p53 normally activate to promote apoptosis via inhibition of bcl-2?
3

Puma
p53 is the most commonly mutated tumor suppressor gene involved in human cancer and is often mutated in SCC
and BCC. p53 acts via two main pathways, 1) activation of p21 (Cdk inhibitor) which leads to cell cycle arrest, and
2) activation of Puma which inhibits Bcl-2 (apoptosis inhibitor) thereby leading to cell death. CKDN2A, a gene that
when mutated leads to a risk of melanoma, acts via 1) activation of p16 (another Cdk inhibitor) and 2) activation of
p14ARF which inhibits Mdm2 (which normally degrades p53). Akt is involved in the PI3K-Akt signaling pathway and
inhibits cell cycle arrest and apoptosis.
Q/Q(M)-482363 Report a Problem
All of the following are true regarding lentigo maligna except:
1

High rates of recurrence
2

Occurs mostly on head and neck
3

Mostly in sun-exposed areas
4

Margins difficult to evaluate
5

Spares oral mucosa
Q/Q(M)-477197 Report a Problem
All of the following are true regarding lentigo maligna except:
5

Spares oral mucosa
Lentigo maligna is a type of melanoma in situ that usually presents as a poorly circumscribed, variably colored
33

patch. It occurs on sun-exposed areas on the head and neck. Margins may be difficult to evaluate and recurrences
are relatively common. There are rare reports of spread onto the oral mucosa as well as conjunctiva.
Q/Q(M)-477197 Report a Problem
A patient having Mohs surgery for a squamous cell carcinoma on the ear has tumor invading the cartilage. What
stage disease does the patient have?
1

T0
2

T1
3

T2
4

T3
5

T4
Q/Q(M)-477190 Report a Problem
A patient having Mohs surgery for a squamous cell carcinoma on the ear has tumor invading the cartilage. What
stage disease does the patient have?
5

T4
The staging of SCCs are as follows. T0 lesions are in situ. T1 lesions are less than 2 cm in diameter. T2 lesions are
2-4 cm in diameter. T3 lesions are greater than 4 cm in diameter. T4 lesions are invasive of muscle, cartilage, or
bone.
Q/Q(M)-477190 Report a Problem
What is the average time between a BCC primary tumor and its metastasis?
1

6 months
2

1 year
3

2-4 years
4

9 years
5

greater than 15 years
Q/Q(M)-482917 Report a Problem
What is the average time between a BCC primary tumor and its metastasis?
4

9 years
The size of a BCC is related to its metastasis risk. Invasion of bone, cartilage, and muscle is not common, however,
34

spread does occur along perichondrium, periosteum, fascia, or tarsal plate.
Q/Q(M)-482917 Report a Problem
What is the most common location for this rapidly growing tumor?
1

Head and neck
2

Chest
3

Back
4

Arms
5

Legs
Q/Q(M)-476841 Report a Problem

What is the most common location for this rapidly growing tumor?
1

Head and neck
Merkel cell carcinoma a cutaneous neuroendocrine carcinoma that usually occurs on the head and neck. It is a
high grade malignant tumor with a 5-year mortality rate of 30-64%. On histopathology, there are trabecular
strands of basophilic cells that stain with a characteristic paranuclear dot pattern with cytokeratin 20.
Q/Q(M)-476841 Report a Problem
An elderly man presents to your office with a telangiectatic, violaceous 1cm dome-shaped nodule on the neck.
Biopsy reveals large, solid nests of cells of intermediate size, with a trabecular pattern at the periphery. These cells
involve the dermis and spread into the subcutaneous fat, but spare the overlying epidermis. The cells are round
and uniform in size, with a round to oval nucleus, small nucleoli, and evenly dispersed chromatin. Numerous
mitotic figures and necrotic areas are seen. Neuron specific enolase is positive. Which of the following is true
regarding this patient's diagnosis?
1

Mohs micrographic surgery is contraindicated in treatment of this lesion
2

Vimentin and desmin stains may be positive
35

3

S-100 stains should be positive
4

This lesion may contain ACTH
5

The neoplasm should not contain gastrin
Q/Q(M)-476106 Report a Problem
An elderly man presents to your office with a telangiectatic, violaceous 1cm dome-shaped nodule on the neck.
Biopsy reveals large, solid nests of cells of intermediate size, with a trabecular pattern at the periphery. These cells
involve the dermis and spread into the subcutaneous fat, but spare the overlying epidermis. The cells are round
and uniform in size, with a round to oval nucleus, small nucleoli, and evenly dispersed chromatin. Numerous
mitotic figures and necrotic areas are seen. Neuron specific enolase is positive. Which of the following is true
regarding this patient's diagnosis?
4

This lesion may contain ACTH
This patient has a Merkel cell carcinoma. Vimentin, desmin, and S-100 are consistently absent in MCC. Mohs
micrographic surgery has been used successfully for the treatment of MCC, with or without adjuvant therapy. This
neoplasm may sometimes contain several neuropeptides including vasoactive intestinal peptide, calcitonin, ACTH,
gastrin, and somatostatin.
Q/Q(M)-476106 Report a Problem
What is the most common location for a fibroepithelioma of pinkus variant of BCC is
1

head and neck
2

extremities
3

hands and feet
4

upper trunk
5

lumbosacral region
Q/Q(M)-482908 Report a Problem
What is the most common location for a fibroepithelioma of pinkus variant of BCC is
5

lumbosacral region
Fibroepithelioma of pinkus variant of BCC is often a fleshy or pink colored nodule. The most common location of a
fibroepithelioma of pinus is the lumbosacral region. Superficial variants are more common on the trunk and
extremities. BCC makes up the most common non melanotic skin cancer at 75%.
Q/Q(M)-482908 Report a Problem
All of the following statements regarding the patient pictured are true EXCEPT:
36

1

Spina bifida may be an associated finding
2

The patient likely has a mutation in the PTCH gene
3

Ameloblastoma is a tumor associated with this disease
4

This patient likely had a similarly affected parent
5

These lesions appeared in childhood
Q/Q(M)-474911 Report a Problem

All of the following statements regarding the patient pictured are true EXCEPT:
3

Ameloblastoma is a tumor associated with this disease
The patient has basal cell nevus syndrome, or Gorlin syndrome, which is characterized by the appearance of
multiple basal cell carcinomas in childhood, odontogenic keratocysts of the jaw, and skeletal defects (i.e
macrocephaly, hypertelorism, frontoparietal bossing, spina bifida, or rib abnormalities). Tumors associated with
this disease include medulloblastoma and meningioma (not ameloblastoma). It is caused by a mutation in the
PTCH gene located on 9q22 and is inherited in an autosomal dominant manner.
Q/Q(M)-474911 Report a Problem
The most common site for intra oral melanoma is?
1

buccal
2

soft palate
3

hard palate
4

gingiva
5

uvula
Q/Q(M)-482205 Report a Problem
The most common site for intra oral melanoma is?
37

3

hard palate
Multiple case series have shown that the hard palate (and specifically the anterior hard palate/alveolar arch) is the
highest risk location for intra-oral melanoma. Prognosis is generally worse for oral melanoma, which is usually due
to delay in diagnosis and presentation with more invasive disease.
Q/Q(M)-482205 Report a Problem
Which of the following are features of dermatofibromas, which help differentiate it from dermatofibrosarcoma
protuberans?
1

Commonly located on the extremities
2

Factor XIIIa negative
3

Factor XIIIa positive
4

Commonly located on the trunk
5

Commonly located on the extremities and factor XIIIa positive
Q/Q(M)-474989 Report a Problem
Which of the following are features of dermatofibromas, which help differentiate it from dermatofibrosarcoma
protuberans?
5

Commonly located on the extremities and factor XIIIa positive
Typically DFSP is CD34 positive and factor XIIIa negative, allowing its differentiation from dermatofibroma.
Q/Q(M)-474989 Report a Problem
The most common location for primary mucinous carcinoma is:
1

neck
2

eyelid
3

areola
4

scrotum
5

nose
Q/Q(M)-482483 Report a Problem
The most common location for primary mucinous carcinoma is:
2

Eyelid
38

Mucinous carcinoma presents as a slowly growing, asymptomatic, round, erythematous nodule on the head and
neck. Forty percent of cases occur on the eyelid. Histologically it is characterized by the presence of large areas of
mucin ( sea of mucous ) with small islands of basophilic epithelial cells. Primary mucinous carcinoma of the
skin has an indolent course. Local recurrence occurs in 1/3 of patients following excision. The rate of metastasis is
low (9.6%).
Q/Q(M)-482483 Report a Problem
What is the most common location of oral SCC?
1

Soft palate
2

Buccal mucosa
3

Gingiva
4

Dorsal tongue
5

Lateral tongue
Q/Q(M)-476592 Report a Problem
What is the most common location of oral SCC?
5

Lateral tongue
Squamous cell carcinomas are the most common carcinoma of the oral cavity. The most common locations for this
tumor are the lateral and ventral surfaces of the tongue and the floor of the mouth.
Q/Q(M)-476592 Report a Problem
Dermoscopic features suggestive of malignant melanoma include:
1

Presence of 2 or more colors within the lesion
2

Blue-whitish veil
3

Asymmetric radial streaming
4

Abrupt interruption of pigment network in the periphery
5

All of these answers are correct
Q/Q(M)-474426 Report a Problem
Dermoscopic features suggestive of malignant melanoma include:
5

All of these answers are correct
Asymmetry, multicomponent pattern, blue-whitish veil, parallel-ridge pattern, atypical pigment network, uneven
39

radial streaming, localized irregular and diffuse pigmentation, irregularly distributed globules, and regression
structures are all dermoscopic features suggestive of malignant melanoma.
Q/Q(M)-474426 Report a Problem
All of the following are risk factors for metastasis from a primary squamous cell carcinoma EXCEPT:
1

Increasing tumor size
2

Location on the ear
3

Recurrent tumor
4

Tumor within ulcer
5

All of these answers are correct
Q/Q(M)-476123 Report a Problem
All of the following are risk factors for metastasis from a primary squamous cell carcinoma EXCEPT:
5

All of these answers are correct
The risk of metastasis from a primary SCC increases with tumor size (>2cm), location (lips, ear, and eyelid, among
others), depth of invasion, immunosuppression, degree of differentiation, and perineural invasion. Recurrent
tumors and SCCs arising in areas of chronic inflammation (such as osteomyelitis, burn scars, or ulcers) are also
considered high risk for metastases.
Q/Q(M)-476123 Report a Problem
Mutations of the p53 gene has been associated with the development of:
1

Mmelanoma
2

Kaposis sarcoma
3

Actinic keratosis
4

Molluscum contagiosum
5

All of these answers are correct
Q/Q(M)-474448 Report a Problem
Mutations of the p53 gene has been associated with the development of:
3

Actinic keratosis
UVB radiation triggers the formation of thymidine dimers both in DNA and RNA, resulting in mutated
keratinocytes. The mutations occur on the tumor suppressor gene p53 within the keratinocytes resulting in
40

impairment of the mechanism of apoptosis. Therefore, clonal expansion of mutated keratinocytes may occur
leading to the formation of AKs.
Q/Q(M)-474448 Report a Problem
Which is the most common neoplasm in patients who have had long-term PUVA therapy?
1

Basal cell carcinoma
2

Atypical fibroxanthoma
3

Squamous cell carcinoma
4

Cutaneous T cell lymphoma
5

Melanoma
Q/Q(M)-477325 Report a Problem
Which is the most common neoplasm in patients who have had long-term PUVA therapy?
3

Squamous cell carcinoma
The most common neoplasm to arise in patients who have been treated with long-term PUVA therapy is
squamous cell carcinomas. In a study by Stern, et.al. a 12.8 fold risk was found for the development of squamous
cell cancers in patients who received high doses than those that received low doses.
Q/Q(M)-477325 Report a Problem
What is the most common location for an epitheloid sarcoma?
1

head and neck
2

proximal extremities
3

hands and forearms
4

lower legs
5

groin and buttocks
Q/Q(M)-482523 Report a Problem
What is the most common location for an epitheloid sarcoma?
3

hands and forearms
Epithelioid sarcoma is a rare soft tissue sarcoma that most commonly develops in young adults, males greater
than females, with a predilection for the distal upper extremities, namely hands and forearms. Most tumors
present as firm-to-hard palpable masses, either in the deep soft tissue or in the dermis. The superficial lesions can
41

present with ulceration. Five year survival and ten year survival rate for patients with epithelioid sarcoma are
approximately 50-70% and 42-55% respectively (Journal of bone and Joint Surgery (Am), 70-A: 862-870, 1988).
Female patients have a more favorable outcome. Proximal lesions have been shown to have worse outcomes
compared to distal lesions.
Q/Q(M)-482523 Report a Problem
Chloroma is a characteristic cutaneous manifestation of:
1

Tuberous sclerosis
2

Sweets syndrome
3

Neurofibromatosis
4

Leukemia
5

Pseudomonas sepsis
Q/Q(M)-477324 Report a Problem
Chloroma is a characteristic cutaneous manifestation of:
4

Leukemia
Chloromas, also termed granulocytic sarcomas, are a localized tumor composed of immature granulocytic cells.
They frequently have a greenish coloration due to the presence of myeloperoxidase and most commonly affect
the bone. The condition most often occurs in patients with acute leukemia of the myeloid type.
Q/Q(M)-477324 Report a Problem
Which of the following is the most important prognostic indicator in a patient with cutaneous lymphoma?
1

Age
2

Primary vs secondary cutaneous
3

Gender
4

Extent of cutaneous involvement
5

Subtype of lymphoma
Q/Q(M)-477176 Report a Problem
Which of the following is the most important prognostic indicator in a patient with cutaneous lymphoma?
2

Primary vs secondary cutaneous
When staging lymphoma, it is critical to determine whether the lymphoma is primary cutaneous arising in the skin
42

or secondary cutaneous arising in association with nodal or extranodal tumor. The prognosis is worse in secondary
when compared to primary lymphomas, irrespective of histologic diagnosis.
Q/Q(M)-477176 Report a Problem
Denileukin diftitox (ONTAK):
1

is a systemic treatment option for CTCL
2

is a diphtheria fusion toxin
3

targets the interleukin-2 receptor
4

None of these answers are correct
5

All of these answers are correct
Q/Q(M)-474430 Report a Problem

Denileukin diftitox (ONTAK):
5

All of these answers are correct
Denileukin diftitox is as diphtheria fusion toxin that targets the IL-2 receptor. It is a systemic treatment alternative
for recalcitrant or advance CTCL.
Q/Q(M)-474430 Report a Problem
All of the following are true regarding curettage of giant congenital melanocytic nevi except:
1

Treatment remains a controversial topic
2

Best performed during the first 2 weeks of life
3

Decreases the risk of melanoma
4

Offers an adequate alternative to surgical excision
5

Mandates careful long-term follow-up
Q/Q(M)-477360 Report a Problem
All of the following are true regarding curettage of giant congenital melanocytic nevi except:
3

Decreases the risk of melanoma
Curettage of giant congenital nevis is a highly controversial treatment option. De Raeve and Roseeuw reported on
16 neonates who underwent curettage for treatment of their giant congenital nevi. They noted that it was best
43

performed within the first two weeks of life. They claim that it offers an adequate cosmetic alternative to surgical
excision. The risk of melanoma developing within the giant congenital nevus is not decreased and long-term
follow-up is essential.
Q/Q(M)-477360 Report a Problem
The Gorlin syndrome is characterized by:
1

Multiple BCCs during childhood
2

Macrocephaly
3

Odontogenic keratocysts of the jaw
4

Autosomal-recessive inheritance pattern
5

All of these answers are correct except Autosomal-recessive inheritance pattern
Q/Q(M)-474416 Report a Problem
The Gorlin syndrome is characterized by:
5

All of these answers are correct except Autosomal-recessive inheritance pattern
Gorlins syndrome is characterized by the appearance of multiple BCCs during childhood, odontogenic
keratocysts of the jaw, and skeletal defects. It is inherited in an autosomal dominant pattern.
Q/Q(M)-474416 Report a Problem
A male teenager had this skin lesion on the oral exam. He did not have any symptoms associated with it. He was
found to have similar lesions on upper extremities. What is the syndrome most likely associated with this finding?
1

Turner syndrome
2

Noonan syndrome
3

Beckwith-Wiedemann syndrome
4

Primary amyloidosis
5

MEN type Iia
Q/Q(M)-482877 Report a Problem
A male teenager had this skin lesion on the oral exam. He did not have any symptoms associated with it. He was
found to have similar lesions on upper extremities. What is the syndrome most likely associated with this finding?
2

Noonan syndrome
This is a granular cell tumor. Patient had several lesions on upper extremities. Biopsy confirmed he had multiple
44

granular cell tumors. Multiple granular cell tumors in kids are associated with Noonan syndrome and
Neurofibromatosis. They can occur anywhere, but more frequently on the tongue, upper extremities and trunk.
Q/Q(M)-482877 Report a Problem
What is the mean time-frame for development of Stewart-Treves Syndrome?
1

Less than 1 year
2

1 year
3

5 years
4

10 years
5

>20 years
Q/Q(M)-482497 Report a Problem
What is the mean time-frame for development of Stewart-Treves Syndrome?
5

>20 years
Stewart-Treves syndrome refers to an angiosarcoma arising from chronic lymphedema. It typically occurs as a
complication of long-lasting lymphedema of the arm after mastectomy and/or radiotherapy for breast cancer. It is
a very late complication of disease, usually developing several years later. The mean time-frame for development
of Stewart-Treves Syndrome in the literature is 23 years (4-40 years).
Q/Q(M)-482497 Report a Problem
Knowing that the patient in this picture has a biopsy confirmed BCC, which of the following would be the
treatment of choice:
1

Conventional excision
2

Imiquimod
3

Radiation
4

Mohs micrographic surgery
5

Cryosurgery
Q/Q(M)-474905 Report a Problem
45


Knowing that the patient in this picture has a biopsy confirmed BCC, which of the following would be the
treatment of choice:
4

Mohs micrographic surgery
Providing the highest cure rates, Mohs micrographic surgery is the treatment of choice for large BCCs (?1cm on
the face or ?2 cms on the trunk), located on high-risk anatomic areas (ear, eyelids, lips, genitals, nose, temples). ,
Morpheaform BCCs, tumors with positive margins after conventional excision, and recurrent BCCs, are also an
indication for Mohs surgery.
Q/Q(M)-474905 Report a Problem

Which of the following is commony seen in seborrheic keratoses when examined with dermoscopy?
1

Maroon lagoons
2

Network
3

Pigment globules
4

Milia like cysts
5

Blue-gray veil
Q/Q(M)-474393 Report a Problem
Which of the following is commony seen in seborrheic keratoses when examined with dermoscopy?
4

Milia like cysts
The main dermoscopic features of seborrheic keratoses include comedolike openings, fissures, milialike cysts,
fingerprinting and lack of true pigment network. In fact, it is the lack of true pigment network, branched streaks,
and pigment globules that differentiate SKs from melanocytic lesions.
Q/Q(M)-474393 Report a Problem
What is the eponym for this metastasis to the umbilicus?
46

1

Tripe palm
2

Sign of Leser-Trelat
3

Pityriasis rotunda
4

Trousseau syndrome
5

Sister Mary Joseph Nodule
Q/Q(M)-476847 Report a Problem

What is the eponym for this metastasis to the umbilicus?
5

Sister Mary Joseph Nodule
Sister Mary Joseph nodule is a cutaneous metastasis that is most commonly associated with stomach, large bowel,
ovary, and pancreatic cancer. Tripe palms are most commonly associated with lung, Leser-Trelat with
adenocarcinomas of the stomach, colon and breast, Trousseau Sign with carcinoma of the pancreas and pityriasis
rotunda with hepatocellular carcinoma and gastric cancer.
Q/Q(M)-476847 Report a Problem
A patient has a malignant melanoma 1.6mm thick with ulceration and a micrometastasis in 1 node. The patient's
staging according to the American Joint Committee on Cancer Staging System is:
1

IIC
2

IIIA
3

IIIB
4

IIIC
5

IV
Q/Q(M)-476101 Report a Problem
A patient has a malignant melanoma 1.6mm thick with ulceration and a micrometastasis in 1 node. The patient's
staging according to the American Joint Committee on Cancer Staging System is:
47

3

IIIB
An ulcerated tumor of any size with micrometastasis in 1 node is T1-4b N1a M0. This corresponds to stage IIIB.
Q/Q(M)-476101 Report a Problem
Desmoplastic trichoepitheliomas are commonly located on:
1

Legs
2

Chest
3

Back
4

Face
5

Scalp
Q/Q(M)-474401 Report a Problem
Desmoplastic trichoepitheliomas are commonly located on:
4

Face
Desmoplastic trichoepithelioma presents as small, firm, umbilicated papule on the face of young adults.
Q/Q(M)-474401 Report a Problem
Sixty-year old, male patient that presents with a stuck on, waxy, hyperkeratotic and hyperpigmented plaque
on his back. The most likely diagnosis is:
1

Actinic keratosis
2

SCC in situ
3

Angiosarcoma
4

Seborrheic keratosis
5

None of these answers are correct
Q/Q(M)-474900 Report a Problem
Sixty-year old, male patient that presents with a stuck on, waxy, hyperkeratotic and hyperpigmented plaque
on his back. The most likely diagnosis is:
4

Seborrheic keratosis
The diagnosis is SK. Notice the warty surface of the lesion with a stuck on appearance.
48

Q/Q(M)-474900 Report a Problem
A patient with multiple skin nodules has a biopsy suggestive of cutaneous lymphoma. The next appropriate step in
making the diagnosis is:
1

Polymerase chain reaction
2

Complete blood count
3

Flow cytometry
4

Immunohistochemical stains
5

Chest xray
Q/Q(M)-477146 Report a Problem
A patient with multiple skin nodules has a biopsy suggestive of cutaneous lymphoma. The next appropriate step in
making the diagnosis is:
1

Polymerase chain reaction
In a patient with skin lesions and pathology suggestive of cutaneous lymphoma, gene rearrangement studies
should be done to detect a clonal population of lymphocytes. Polymerase chain reaction (PCR) is approximately
1000 times more sensitive than Southern blot in the detection of a clonal population of cells. In addition, PCR may
be used on formalin fixed tissue as well as fresh tissue, whereas Southern blot testing must be done of fresh tissue
only.
Q/Q(M)-477146 Report a Problem
What would you expect to see under dermoscopy of this vascular neoplasm?
1

Red sacculae
2

Arborizing blood vessels
3

Hair pin telangectasia
4

Milky red globules
5

Blue-grey ovoid nests
Q/Q(M)-476849 Report a Problem
49


What would you expect to see under dermoscopy of this vascular neoplasm?
1

Red sacculae
Hemangioma have a characteristic appearance under dermoscopy. Typically, they have a maroon lagoon or red
sacculae appearance.
Q/Q(M)-476849 Report a Problem

The human papilloma virus type associated with red brown smooth and warty papules is
1

HPV 1
2

HPV 5
3

HPV 7
4

HPV 13
5

HPV 16
Q/Q(M)-476654 Report a Problem
The human papilloma virus type associated with red brown smooth and warty papules is
5

HPV 16
Bowenoid papulosis manifests clinically as multiple red-brown warty papules or confluent planques on the
external genitalia. These lesions may resemble genital warts but histologically represent high-grade squamous
intraepithelial lesions. Bowenoid papulosis is caused by infection by HPV and linked to HPV 16, 18, 31, 35, and 39.
Q/Q(M)-476654 Report a Problem
50

40 year-old male presented with a slow growing large tumor on the back. The patient undergoes wide excision
with adjunctive chemotherapy. Which of the chemotherapeutic agents was used?
1

Imatinib
2

Cytarabine
3

Doxorubicin
4

Vincristine
5

Cisplatin
Q/Q(M)-478125 Report a Problem
40 year-old male presented with a slow growing large tumor on the back. The patient undergoes wide excision
with adjunctive chemotherapy. Which of the chemotherapeutic agents was used?
1

Imatinib
Imatinib or Gleevec is used in the treatment of primary or locally recurrent dermatofibrosarcoma protuberans.
Even with wide excision, the 5 year recurrence rate for this tumor can be 20-25%. Activation of the Platelet
derived growth factor receptor associated with overexpression of Platelet Derived growth factor is central to the
development of DFSP Imatinib is a protein tyosine kinase inhibitor for Bcr-Abl kinase and is used primary in chronic
myelogenous leukemia with the Philadelphia chromosome defect. Imatinib also inhibits the typosine kinases
associated with platelet derived growth factor and stem cell factor. Therefore, it directly inhibits the Platelet
Derived Growth factor receptor signaling cascade,that plays a critical role in the pathogenesis and growth of
DFSPs. While Imatinib has been successful in clinical trials, it is not yet FDA approved for the treatment of DFSPs.
The other listed agents are not adjunctive or primary therapies for DFSP.
Q/Q(M)-478125 Report a Problem
Characteristics indicative of a high risk of metastasis with SCC include:
1

Mitotic rate
2

Extremity location
3

Well differentiated
4

Deep invasion
5

Papillomavirus infection
Q/Q(M)-478728 Report a Problem
Characteristics indicative of a high risk of metastasis with SCC include:
4

Deep invasion
51

The rate of SCC metastasis from all skin sites ranges from 0.5% to 5.2%. Local recurrences and metastasis are
related to: 1.treatment modality, 2. prior hitological, 3. location, 4. size, 5.depth, 6. histological differentiation, 7.
histological evidence of perineural involvement, 8. histological evidence of desmoplastic features, 9. precipitating
factors other than UV light, and 10. host immunosuppression. In reference to metastatic disease, the highest rates
occur from scars, the lip, and the external ear. Patients with perineural spread have a local recurrence rate of
47.2% and a metastatic rate of 34.8%. Demosplastic SCC's are 6 times more likely to metastasize than other
histological patterns.
Q/Q(M)-478728 Report a Problem
The patient is a 45 year old male complaining of red, chapped lower lip. Which of the following lasers is the most
appropriate to treat this condition?
1

Pulsed Dye Laser
2

Nd:YAG laser
3

CO2 laser
4

Diode laser
5

Laser treatment is not an option
Q/Q(M)-474901 Report a Problem

The patient is a 45 year old male complaining of red, chapped lower lip. Which of the following lasers is the most
appropriate to treat this condition?
3

CO2 laser
The patient has actinic cheilitis. Notice the red, scaly lower lip, with erosions and fissures. The CO2 laser is
currently a common treatment alternative for this condition.
Q/Q(M)-474901 Report a Problem
A patient with a innumerable disseminated keratoacanthomas, including lesions on the larynx and oral mucosa:
1

Is unlikely to have palmoplantar involvement
52

2

Likely has an underlying immune deficiency
3

Is at high risk for myelodysplasia
4

Likely inherited their condition in an autosomal dominant manner
5

Likely developed them during adulthood
Q/Q(M)-476098 Report a Problem
A patient with a innumerable disseminated keratoacanthomas, including lesions on the larynx and oral mucosa:
5

Likely developed them during adulthood
This patient has the Grzybowski type of keratoacanthomas. Typically diagnosed in adulthood, these patients have
the sudden appearance of hundreds of small lesions in a disseminated fashion. The lesions can be found anywhere
on the body including palms, soles, larynx, and oral mucosa.
Q/Q(M)-476098 Report a Problem
All of the following statements regarding Bowen's disease are true EXCEPT:
1

Lesions arising on the lower limbs are more common in men than in women
2

The basement membrane remains intact on histopathology
3

5% of patients with Bowen's disease develop invasive squamous cell carcinoma
4

Chronic sun exposure is a risk factor for Bowen's disease
5

Mucosal Bowen's disease may appear as a verrucous plaque
Q/Q(M)-476125 Report a Problem
All of the following statements regarding Bowen's disease are true EXCEPT:
1

Lesions arising on the lower limbs are more common in men than in women
The most common locations for BD include the head and neck regions and the extremities. BD arising on the lower
limbs is frequently found in women, whereas lesions located on the ears and scalp are more common in men. The
basement membrane remains intact on histopathology. 5% of patients develop invasive SCC. Risk factors include
chronic sun exposure, immunosuppression, HPV, arsenic exposure, and ionizing radiation. Mucosal BD may appear
as a verrucous or polypoid plaque, as an erythroplakic patch, or as a velvety red plaque.
Q/Q(M)-476125 Report a Problem
A 60 y/o male smoker with multiple sclerosis presents for evaluation of painful 5cm x 4cm plaque on right
posterior shoulder, which appeared over 3 months. What is the most likely diagnosis?
53

1

Irritated Seborrheic Keratosis
2

Squamous Cell Carcinoma
3

Basal Cell Carcinoma
4

Metastatic Carcinoma
5

Malignant Melanoma
Q/Q(M)-482886 Report a Problem
A 60 y/o male smoker with multiple sclerosis presents for evaluation of painful 5cm x 4cm plaque on right
posterior shoulder, which appeared over 3 months. What is the most likely diagnosis?
3

Basal Cell Carcinoma
Basal cell carcinoma. It has a male to female ratio 2:1. UVB damages DNA and produces a C-T transition mutation
(65% of BCC). P53 and PTCH genes are commonly affected. It has a slow, indolent growth potential: metastatic
potential 0.0028% to 0.1%.; known to metastasize to lymph nodes and lungs.
Q/Q(M)-482886 Report a Problem
Which of the following markers do not stain melanocytic lesions:
1

Neuron specific enolase
2

Vimentin
3

S-100
4

HMB-45
5

All of these answers are correct
Q/Q(M)-474449 Report a Problem
Which of the following markers do not stain melanocytic lesions:
1

Neuron specific enolase
Neuron specific enolase is positive in Merkel cell carcinoma. Vimentin, S-100 and HMB-45 stains melanocytic
lesions.
Q/Q(M)-474449 Report a Problem
Which of the following best describes the incidence of skin cancer in transplant recipients from greatest to least?
1

BCC>melanoma>SCC>Merkel cell
54

2

SCC>BCC>melanoma>Merkel cell
3

BCC>SCC>melanoma>Merkel cell
4

SCC>BCC>Merkel cell>melanoma
5

Merkel cell>SCC>melanoma>BCC
Q/Q(M)-478725 Report a Problem
Which of the following best describes the incidence of skin cancer in transplant recipients from greatest to least?
2

SCC>BCC>melanoma>Merkel cell
The incidence of skin cancer in transplant recipient (from greatest to least) is as follows:
SCC>BCC>melanoma>Merkel cell.
Q/Q(M)-478725 Report a Problem
Histologically, this lesion is shows plump, polygonal cells arranged in nests and fascicles with granular cytoplasm.
Which immunohistochemical stain would be positive?
1

Colloidal iron
2

Von kossa
3

Warthin-starry
4

S-100
5

CD68
Q/Q(M)-476839 Report a Problem

Histologically, this lesion is shows plump, polygonal cells arranged in nests and fascicles with granular cytoplasm.
Which immunohistochemical stain would be positive?
55

4

S-100
Granular cell tumors are benign growths which typically occur on the tongue. They are typically well-
circumscribed, raised, firm nodules. Histologically, the cells have uniform nuclear characteristics and granular
cytoplasm due to presence of lysozyme. The lesions are PAS positive and S-100 positive.
Q/Q(M)-476839 Report a Problem
What is the diagnosis?
1

Condyloma acuminata
2

Molloscum contagiosum
3

Pearly penile papules
4

Lichen planus
5

Psoriasis
Q/Q(M)-476845 Report a Problem

What is the diagnosis?
3

Pearly penile papules
Pearly penile papules are benign, filiform papules that form along the corona of the penis. THey are often
mistaken for a form of veneral disease. Histologically, they are similar to angiofibromas and show ectatic vessels
surrounded by dense connective tissue.
Q/Q(M)-476845 Report a Problem
A 55 year-old female presents with an ulcerated malignant melanoma with Breslow dept of 1.5mm. Sentinel
lymph node biopsy is negative. Which of the following is the correct staging classification?
1

Stage IB
2

Stage IIA
3

Stage IIB
56

4

Stage IIIA
5

Stage IIIB
Q/Q(M)-477399 Report a Problem
A 55 year-old female presents with an ulcerated malignant melanoma with Breslow dept of 1.5mm. Sentinel
lymph node biopsy is negative. Which of the following is the correct staging classification?
2

Stage IIA
The current melanoma staging is developed by the American Joint Comittee on Cancer. This system is based on
four characterisitics: 1. Tumor thickness 2.Presence of ulceration within the primary tumor, 3. Involvement of
lymph nodes and 4.presence of distal metastases. Stage 1A characterizes a melanoma that has less than 1mm in
tumor thickness and no ulceration, no nodes, and no distant mets. Stage IB characterizes a melanoma with tumor
thickness less than 1mm however with ulceration and no nodes or distal metastases. Stage IB also can characterize
melanomas with tumor thickness between 1.01-2.00 mm without ulceration. Stage IIA classifies tumor with
thickness between 1.01-2.00mm, with ulceration, and no nodes and no distal metastases. Stage IIA also
characterizes those melanomas with thickness between 2.01-4.0mm without ulceration, nodes, or distal mets.
Stage III and IV melanomas have nodal and distal met involvement respectively.
Q/Q(M)-477399 Report a Problem
Which neoplasm is associated with the Stewart-Treves syndrome?
1

Renal leiomyomas
2

Basal cell carcinoma
3

Angiosarcoma
4

Keratoacanthoma
5

T cell lymphoma
Q/Q(M)-477129 Report a Problem
Which neoplasm is associated with the Stewart-Treves syndrome?
3

Angiosarcoma
Stewart-Treves syndrome is the development of angiosarcoma in the setting of chronic lymphedema. Originally,
named after radical mastectomy for the treatment of breast cancer. The term applies to the development of
angiosarcoma in any chronic lymphedematous condition.
Q/Q(M)-477129 Report a Problem
Mutations in which gene would likely be found in the neoplastic cells of this lesion?
57

1

PATCH
2

p53
3

Fumarate hydratase
4

CREBBP
5

p63
Q/Q(M)-476653 Report a Problem

Mutations in which gene would likely be found in the neoplastic cells of this lesion?
2

p53
Squamous cell carcinoma is the second most common cancer of the skin. Mutations in the tumor suppressor p16
and p53 are commonly found in SCC's. Normally, UV damage upregulates p53 thereby delaying cell cycle
progression. DNA damage can then be repaired or the cell could undergo apoptosis. In squamous cell carcinoma,
p53 exhibits loss of heterozygosity due to C to T or CC to TT mutations.
Q/Q(M)-476653 Report a Problem
All of the following may increase the incidence of SCC, EXCEPT:
1

Sun exposure
2

Immunosuppression
3

Increasing age
4

Proximity to the equator
5

All of the following may increase the incidence of SCC
Q/Q(M)-474434 Report a Problem
All of the following may increase the incidence of SCC, EXCEPT:
5

All of the following may increase the incidence of SCC
58

The incidence of SCC increases with age, and it is 35 times higher in individuals older than 75 years of age when
compared to ages 50-55. The incidence doubles for each 8 to 10 degree decline in latitude, therefore, individuals
living closer to the equator have a greater risk.
Q/Q(M)-474434 Report a Problem

The incidence of cutaneous squamous cell carcinoma in organ transplant recipients is increased by how much
compared with the general population?
1

2 fold
2

5 fold
3

10 fold
4

20 fold
5

65 fold
Q/Q(M)-477368 Report a Problem
The incidence of cutaneous squamous cell carcinoma in organ transplant recipients is increased by how much
compared with the general population?
5

65 fold
The risk of skin cancer in organ transplant patients is dramatically increase and may be more aggressive. In one
study, SCCs had a 65x increased incidence, BCC 10x, and melanoma 3.4x.
Q/Q(M)-477368 Report a Problem
Which of the following statements regarding prognosis for malignant melanoma is true?
1

Increasing age has a positive effect on survival.
2

Patients with primary lesions located on the extremities (except acral lesions) have a worse prognosis than
those with tumors located on the trunk.
3

There is a higher survival rate for patients with palpable metastatic nodes compared to those with
micrometastatic nodal disease.
4

For stage IV disease, patients with non-visceral metastases (eg skin, subcutis, distant lymph nodes) have a
better prognosis compared with those with visceral metastases.
5

Male gender has a positive effect on survival.
Q/Q(M)-476727 Report a Problem
Which of the following statements regarding prognosis for malignant melanoma is true?
59

4

For stage IV disease, patients with non-visceral metastases (eg skin, subcutis, distant lymph nodes) have a
better prognosis compared with those with visceral metastases.
Increasing age and male gender have a negative effect on survival in melanoma. Patients with primary lesions
located on the extremities (except acral lesions) have a better prognosis than those with tumors located on the
head, neck, or trunk. There is a significantly lower survival for patients with palpable metastatic nodes
(macrometastasis) when compared to those with micrometastatic nodes. For stage IV disease, patients with non-
visceral metastases (eg skin, subcutis, distant lymph nodes) have a better prognosis compared with those with
visceral metastases.
Q/Q(M)-476727 Report a Problem
Which of the following is a clinical stage of keratoacanthomas?
1

Plaque
2

Mature
3

Proliferative
4

Resolving
5

All of these answers are correct except plaque
Q/Q(M)-474445 Report a Problem
Which of the following is a clinical stage of keratoacanthomas?
5

All of these answers are correct except plaque
Keratoacanthomas have three, consecutive, clinical stages: proliferative, mature and resolving. The proliferative
stage is characterized by the appearance of a rapid growing papule. This phase is followed by the mature stage
when the lesion acquires its characterisitic dome-shaped appearance with a central, keratinous core. Tumor
resorption occurs during the involution stage resulting in a slightly depressed, hypopigmented scar.
Q/Q(M)-474445 Report a Problem
Which of the following melanoma subtypes is more consistently seen in dark-skinned individuals?
1

Acral lentiginous melanoma
2

Superficial spreading melanoma
3

Lentigo maligna melanoma
4

Nodular melanoma
5

Superficial spreading melanoma and Lentigo maligna melanoma
60

Q/Q(M)-474428 Report a Problem

Which of the following melanoma subtypes is more consistently seen in dark-skinned individuals?
1

Acral lentiginous melanoma
Acral lentiginous melanoma is the predominant type of melanoma in dark-skinned individuals. It is usually located
on the soles, palms, and subungeal region of patients in their fifth to sixth decade of life.
Q/Q(M)-474428 Report a Problem
What is the most location for this lesion which lacks phosphorylase in the epidermal cells?
1

Back
2

Buttock
3

Leg
4

Neck
5

Sacrum
Q/Q(M)-476836 Report a Problem

What is the most location for this lesion which lacks phosphorylase in the epidermal cells?
3

Leg
Clear cell acanthomas are slow growing, benign growths which typically occur on the leg. They have a striking
histologic appearance in the epidermis with sharp demaracation and enlarged, pale cells. Excess glycogen in the
cells accounts for their clear appearance and is due to a defect in phosphorylase.
Q/Q(M)-476836 Report a Problem
61

A child presents with a giant congenital nevus overlying the back of the skull, extending onto the shoulders. It is
~15% body surface area, sparing the face and anterior neck. Which test should be ordered?
1

A head CT
2

A head MRI
3

A skull plain film X-ray
4

A skin biopsy
5

A PET scan
Q/Q(M)-480184 Report a Problem
A child presents with a giant congenital nevus overlying the back of the skull, extending onto the shoulders. It is
~15% body surface area, sparing the face and anterior neck. Which test should be ordered?
2

A head MRI
Giant congenital nevi overlying the spinal columna nd skull can be associated with neurocutaneous melanosis.
Presenting symptoms include increased cranial pressure, spinal cord compression or leptomeningeal melanoma. A
MRI should be performed in these children to rule-out CNS involvement. The other tests would not be as useful in
this situation.
Q/Q(M)-480184 Report a Problem
Which of the following is true regarding actinic keratoses?
1

Salicylic acid, tretinoin, and alpha-hydroxy acids are not useful in treatment
2

Low fat diets may decrease the incidence of AKs
3

Cure rates for treatment with topical 5-fluorouracil are higher than for treatment with cryotherapy
4

UVA from sunlight is most responsible for AK development
5

Male gender is not a risk factor for AKs.

Which of the following is true regarding actinic keratoses?
2

Low fat diets may decrease the incidence of AKs
Salicylic acid, tretinoin, and alpha-hydroxy acids have been reported to be useful in treatment of AKs.
Low fat diets have been reported to be useful in decreasing the incidence of AKs. Cryotherapy has a
reported cure rate of 98.8% for AKs, whereas topical 5-FU has a reported cure rate of 93%. UVB, not
UVA, is most responsible for AK development. Male gender is a risk factor for the development of AKs.

What is the most common location for pagetoid reticulosis?
1

Trunk
2

Head and neck
3

Hands and feet
62

4

Flexural sites of upper and lower extremities
5

Genitals
What is the most common location for pagetoid reticulosis?
3

Hands and feet
Pagetoid reticulosis is an indolent cutaneous T-cell lymphoma. Pagetoid reticulosis favors an acral
distribution and typically presents as scaly oval plaques. Epidermotropism is present, with tumor cells
being CD4 positive or CD8 positive. Another interesting immunohistochemical finding is the absence of
CD45 expression.

At what location is this tumor LEAST likely to recur after surgical excision?
1

Nasolabial fold
2

Lateral canthus
3

Mid forehead
4

Preauricular
5

Scalp
At what location is this tumor LEAST likely to recur after surgical excision?
3

Mid forehead
Basal cell carcinomas are slow-growing, epithelial tumors. Highest area of recurrences after surgical
excision is in the "H-zone" of the face which include nose, nasolabial folds, periocular, periauricular and
the scalp.
Which of the following is true regarding digital HPV-associated squamous cell cancers?
1

The rate of metastasis approaches 15%.
2

HPV18 is the most common associated sybtype.
3

Mohs micrographic surgery yields a 20% recurrence rate.
4

Women outnumber men 2:1.
5

These lesions only occur in association with immunosuppression.
Which of the following is true regarding digital HPV-associated squamous cell cancers?
3

Mohs micrographic surgery yields a 20% recurrence rate.
According to Riddel et al (JAAD 2011;64(6):1147-1153), Mohs micrographic surgery, although the
treatment of choice, results in a 20% recurrence rate, which is significantly higher than cutaneous SCC.
HPV16 is most often implicated. Men outnumber women 2:1. The rate of metastasis averages between 2-
3%. Although common in transplant patients, HPV associated digital SCCs can occur secondary to trauma
and in immunocompetent patients.
Histopathological evidence of epidermotropism may be seen in which of the following conditions:
1

Merkel cell carcinoma
2

SCC
3

CTCL
4

None of these answers are correct
5

All of these answers are correct
Histopathological evidence of epidermotropism may be seen in which of the following conditions:
3

CTCL
63

Epidermotropism, defined as the presence of lymphocytes in the epidermis is characteristic of CTCL.
Merkel cell carcinoma should be treated with what size surgical margins?
1

2mm
2

5mm
3

1cm
4

3cm
5

5cm
Merkel cell carcinoma should be treated with what size surgical margins?
4

3cm
Merkel cell carcinoma is an aggresive rare tumor of the skin accounting for less than 1% of cutaneous
malignancies. Also known as neuroendocrine cancer of the skin, this tumor presents as a painless red to
violaceous, firm, solitary, nodule that usually presents on sun exposed areas such as the head, neck and
upper extremities. These tumors present ussualy during the 6th and 7th decades, andhave a 2 year survival
rate of 50-70%. Becuase of this tumors high potenital for regional and distal metastasis, thistumor should
be excised with wide local excision with 3cm surgical margins.
The Grzybowski type of keratoacanthoma:
1

Is characterized by rapid growth of a single lesion reaching a diameter of 9 cm or more
2

Typically invades underlying cartilage
3

Demonstrates simultaneous central healing
4

Presents in childhood on sun-exposed surfaces
5

Presents with hundreds of disseminated lesions
The Grzybowski type of keratoacanthoma:
5

Presents with hundreds of disseminated lesions
The Grzybowski type of keratoacanthoma is typically diagnosed in adulthood, with the sudden appearance
of hundreds of lesions in a disseminated fashion. The lesions are generally 2-3mm in diameter and can be
found anywhere on the body including the palms, soles, larynx, and oral mucosa.
Commonly used immunohistochemical markers that are positive in Merkel cell carcinomas include all of
the following EXCEPT:
1

Chromogranin A/B
2

Synaptophysin
3

Glial fibrillary acidic protein
4

Cytokeratin 20
5

Cytokeratin 8
Commonly used immunohistochemical markers that are positive in Merkel cell carcinomas include all of
the following EXCEPT:
3

Glial fibrillary acidic protein
The most commonly used markers for MCC are monoclonal antibodies to cytokeratins 8, 18, and 20;
neuron-specific enolase (the most constant marker); chromogranin A/B, and synaptophysin. Leukocyte
common antigen, vimentin, desmin, glial fibrillary acidic protein, and S-100 are consistently absent in
MCC.
Topical 5-Fluorouracil:
1

Interferes with the synthesis of DNA and RNA
64

2

Is an alternative for the treatment of actinic keratosis
3

May cause pruritus and burning at the site of application
4

All of these answers are correct
5

None of these answers are correct
Topical 5-Fluorouracil:
4

All of these answers are correct
Topical 5-FU blocks the methylation reaction of deoxyuridylic acid to thymidylic acid, thus interfering
with the synthesis of DNA and RNA. Normal side effects during treatment include pruritus and burning at
the site of application. It is a treatment option for patients with multiple AKs within an area.
Which of the following is most likely to present with cutaneous metastases in men?
1

Lung cancer
2

Colon cancer
3

Prostate cancer
4

Melanoma
5

Esophageal Cancer
Which of the following is most likely to present with cutaneous metastases in men?
4

Melanoma
The most common malignancy to present with cutaneous metastases in men is from melanoma, followed
by squamous cell cancer of the head and neck. In women, breast cancer is most likely to present with
cutaneous metastases. Special presentations of cutaneous metastases include alopecia neoplastica (scarring
alopecia) and zosteriform metastases.
The mucin found in this lesion is produced by:
1

Fibroblasts
2

Keratinocyte
3

Merkel cells
4

Nail matrix
5

Osteoblast
1

Fibroblasts
Digital mucous cysts are pseudocysts found on the dorsal digits between the distal interphalangeal joint
and the nail fold. The fibroblasts in digital mucous cysts produce large quantities of hyaluronic acid.
Sentinel lymph node biopsy in malignant melanoma:
1

Has gained acceptance for the treatment of MM of intermediate thickness (1-4mm)
2

Is mainly being used for a more accurate staging
3

Is not recommended
4

None of these answers are correct
5

Has gained acceptance for the treatment of MM of intermediate thickness (1-4mm) and is mainly being
used for a more accurate staging
65

Q/Q(M)-474441 Report a Problem
Sentinel lymph node biopsy in malignant melanoma:
5

Has gained acceptance for the treatment of MM of intermediate thickness (1-4mm) and is mainly being
used for a more accurate staging
Sentinel lymph node biopsy has gained acceptance for the treatment of melanomas 1-4 mm in thickness. Even
though it has been widely accepted due to its low morbidity and high feasibility, the role of SLN biopsy in survival
has not been established. It is mainly being used for a more accurate staging, prognosis and to determine if further
adjuvant therapy is necessary.
Q/Q(M)-474441 Report a Problem
An elderly white man presents with a slowly enlarging, well-demarcated pink, scaly plaque on the neck. Biopsy of
the lesion reveals epidermal dysplasia and keratinocytic disorganization with preservation of the basement
membrane. Hyperkeratosis and parakeratosis are also present, and numerous atypical keratinocytes are seen
throughout the epidermis, with loss of polarity, atypia, and mitoses. A chronic inflammatory infiltrate is present in
the upper dermis. Which of the following treatment(s) should be considered for this patient?
1

Conventional excision
2

Mohs micrographic surgery
3

Imiquimod
4

Conventional excision and Mohs micrographic surgery
5

All of these answers are correct
Q/Q(M)-476479 Report a Problem
An elderly white man presents with a slowly enlarging, well-demarcated pink, scaly plaque on the neck. Biopsy of
the lesion reveals epidermal dysplasia and keratinocytic disorganization with preservation of the basement
membrane. Hyperkeratosis and parakeratosis are also present, and numerous atypical keratinocytes are seen
throughout the epidermis, with loss of polarity, atypia, and mitoses. A chronic inflammatory infiltrate is present in
the upper dermis. Which of the following treatment(s) should be considered for this patient?
5

All of these answers are correct
The patient has Bowen's Disease, or squamous cell carcinoma in situ. Multiple treatment options are available,
including conventional excision, Mohs micrographic surgery, imiquimod, cryosurgery, and 5-FU, among others.
Conventional excision is the most commonly used method. With this method, not only is the lesion removed, but
it also provides the specimen for histologic verification to rule out invasive SCC. Recurrence rates of 5% have been
reported with conventional excision. Mohs micrographic surgery is an option when the lesion is located in areas
where there is an increased risk of sub-clinical spreading, or when tissue sparing is a priority. Imiquimod has
recently been successfully used for the treatment of SCCIS, though standardized regimens have not been
established.
66

Q/Q(M)-476479 Report a Problem
Merkel cell carcinoma has been found to be associated with which of the following viruses?
1

Herpes virus
2

Polyomavirus
3

Paramyxovirus
4

Flavivirus
5

Enterovirus
Q/Q(M)-482256 Report a Problem
Merkel cell carcinoma has been found to be associated with which of the following viruses?
2

Polyomavirus
Merkel cell carcinoma has been found to be associated with the merkel cell polyomavirus (MCPyV). Herpes virus
causes diseases such as HSV, VZV, EBV, CMV, roseola, and Kaposi's sarcoma. Paramyxovirus causes measles.
Flavivirus causes diseases such as west nile virus, dengue, and yellow fever. Enteroviruses are not associated with
merkel cell carcinoma.
Q/Q(M)-482256 Report a Problem
Which of the following are the most reliable prognostic factors in malignant melanoma?
1

Breslows depth and ulceration
2

Breslows depth and Clark level
3

Clark level and ulceration
4

Breslows depth and Clark level
5

None of these answers are correct
Q/Q(M)-474442 Report a Problem
Which of the following are the most reliable prognostic factors in malignant melanoma?
1

Breslows depth and ulceration
The most reliable prognostic factors in MM are Breslows depth and ulceration of the primary tumor. Breslow
depth is the thickness of the melanoma measured from the granular layer to the deepest point of tumor invasion.
Q/Q(M)-474442 Report a Problem
67


Seventy-five year old, male patient, with a 3cms pink, pearly nodule on his chest. You notice that the patient has a
scar on the same area. The treatment of choice is:
1

Radiation
2

Conventional excision
3

Cryotherapy
4

Mohs surgery
5

Imiquimod
Q/Q(M)-475384 Report a Problem
Seventy-five year old, male patient, with a 3cms pink, pearly nodule on his chest. You notice that the patient has a
scar on the same area. The treatment of choice is:
4

Mohs surgery
The most likely diagnosis in this patient is recurrent BCC, therefore Mohs surgery is the treatment of choice.
Furthermore, any NMSC measuring 3 cms located on any anatomical site is an indication for Mohs surgery.
Q/Q(M)-475384 Report a Problem
The risk of a melanoma developing in a giant congenital melanocytic nevus is approximately:
1

3%
2

6%
3

12%
4

50%
5

Virtually all of these patients will develop melanoma
Q/Q(M)-480183 Report a Problem
The risk of a melanoma developing in a giant congenital melanocytic nevus is approximately:
2

6%
Large/giant congenital nevi are greater than 20 cm or greater than 10% of the body surface area. The risk of
melanoma in this type of lesion is approximately 6% by the age of 60 (Rhodes, 1981; Bett, 2005). 50% of these
melanomas occur by 5 years of age.
68

Q/Q(M)-480183 Report a Problem
On histologic examination of the adenocarcinoma of the perineal area, which of the following stains would NOT be
positive?
1

PAS
2

Mucicarmine
3

CEA
4

EMA
5

HMB45
Q/Q(M)-476848 Report a Problem

On histologic examination of the adenocarcinoma of the perineal area, which of the following stains would NOT be
positive?
5

HMB45
Extramammary Paget's cutaneous adenocarcinoma of glandular differentiation. Approximately 25% of these
tumors are associated underlying neoplasms. Histologically, the Paget cells stain positively with PAS, mucicarmine,
CEA, EMA, LMW keratin. They are HMB45 negative which is a melanocytic marker.
Q/Q(M)-476848 Report a Problem
Which of the following is not considered a high-risk location of SCC:
1

Eyelids
2

Nose
3

Ear
4

Lips
5

Chest
69

Q/Q(M)-474447 Report a Problem
Which of the following is not considered a high-risk location of SCC:
5

Chest
High risk locations for recurrence and metastases include the H-zone of the face, skin overlying cartilage and
bony structures (e.g. preauricular area, retroauricular sulcus, nasolabial fold, inner canthus, philtrum, temple,
upper lip, columella, nose, lower eyelid). Lesions located on the trunk and extremities are usually considered low
risk unless aggressive histologic pattern is present.
Q/Q(M)-474447 Report a Problem
What virus is most closely associated with these lesions in this HIV infected patient?
1

Human herpes virus 2
2

Human herpes virus 6
3

Human herpes virus 8
4

Cytomegalovirus
5

Ebstein Barr virus
Q/Q(M)-476684 Report a Problem

What virus is most closely associated with these lesions in this HIV infected patient?
3

Human herpes virus 8
Kaposi's sarcoma is an AIDS defining illness. It if found at a much higher incidence in homosexual men. Human
herpes virus 8 is thought to be pathogenic in this tumor.
70

Q/Q(M)-476684 Report a Problem
Bazex syndrome can be differentiated clinically from Rombo syndrome by presence of
1

Multiple basal cell carcinomas
2

Trichiepitheliomas
3

Milia
4

Bollicular atrophoderma
5

Hypohidrosis
Q/Q(M)-482115 Report a Problem
Bazex syndrome can be differentiated clinically from Rombo syndrome by presence of
5

Hypohidrosis
Localized hypohidrosis is a feature found in Bazex syndrome but not Rombo syndrome. Another differentiating
feature is that Rombo syndrome classically has vermiculate atrophoderma, while Bazex has follicular
atrophoderma.
Q/Q(M)-482115 Report a Problem
Which site of squamous cell carcinoma has the greatest risk of metastasis?
1

lip
2

ear
3

eyelid
4

trunk
5

nose
Q/Q(M)-482907 Report a Problem

Which site of squamous cell carcinoma has the greatest risk of metastasis?
1

Lip
The lip has an approximate rate of metastasis of 13%, which is slightly higher than the metastasis rate of 11% from
the ear. Risks for metastasis include size greater than 2cm, perineural invasion, immunosuppression, treatment
history, degree of tumor differentiation, or location on the ear or lip.
71

Q/Q(M)-482907 Report a Problem
All of the following are true of melanoma-associated leukoderma except:
1

Lesions resemble vitiligo
2

Portends a worse prognosis
3

Seen in patients with metastatic disease but no primary lesion
4

Histology resembles that of a halo nevus
5

Lesions develop distant to melanoma
Q/Q(M)-477354 Report a Problem
All of the following are true of melanoma-associated leukoderma except:
2

Portends a worse prognosis
Hypomelanosis associated with melanoma most commonly may be seen in three ways. The first type is analogous
to a halo nevus. The second is a remote leukoderma distant from the primary lesion. The third is a vitiligo like
leukoderma. It may portend a better prognosis in comparison to others with the same stage of disease.
Q/Q(M)-477354 Report a Problem
Which of the following is true regarding poikilodermatous mycosis fungoides?
1

Majority of cases are predominantly CD8(+)
2

Patients typically have a later age of onset compared to classic mycosis fungiodes
3

Can be associated with LyP
4

More women than men affected
5

Patients typically do not respond well to phototherapy
Q/Q(M)-482499 Report a Problem

Which of the following is true regarding poikilodermatous mycosis fungoides?
3

Can be associated with LyP
While there is an overrepresentation of CD8+ cases compared to classic mycosis fungoides, more cases are still
predominantly CD4+. Patients typically present at a younger age (median age 44 years), with a slight male
predominance. There is an excellent response to phototherapy with clinical improvement in ~90% of patients
treated. There is an increased association with LyP compared to other types of mycosis fungoides. (Abbott et al,
72

JAAD 2011; in press)
Q/Q(M)-482499 Report a Problem
A 65 year-old female with multiple actinic keratosis on the face under treatment with 5-FU. According to the
image and aforementioned information, you may conclude that:
1

The patient must immediately stop treatment since unexpected side effects have developed
2

The patient has been compliant with 5-FU treatment and the appearance of inflammation, erythema and
erosions are expected
3

The image is not relevant to 5-FU treatment
4

None of these answers are correct
5

All of these answers are correct
Q/Q(M)-474904 Report a Problem

A 65 year-old female with multiple actinic keratosis on the face under treatment with 5-FU. According to the
image and aforementioned information, you may conclude that:
2

The patient has been compliant with 5-FU treatment and the appearance of inflammation, erythema and
erosions are expected
Compliance is a key feature in treatment with 5-FU. Erythema, inflammation and erosion must develop and is
considered a sign of successful treatment.
Q/Q(M)-474904 Report a Problem

The cure rate of cryotherapy as a treatment for actinic keratoses is:
1

80%
2

85%
3

90%
73

4

95%
5

99%
Q/Q(M)-476108 Report a Problem

The cure rate of cryotherapy as a treatment for actinic keratoses is:
5

99%
Cryotherapy is the most common treatment for AKs, with a cure rate of 98.8%.
Q/Q(M)-476108 Report a Problem
Keratoacanthomas have been linked etiologically to:
1

Ultraviolet exposure
2

Human papilloma virus
3

Chemical carcinogens such as tar and pitch
4

Smoking
5

All of these answers are correct
Q/Q(M)-476100 Report a Problem
Keratoacanthomas have been linked etiologically to:
5

All of these answers are correct
The origin of KAs has not been established. Ultraviolet exposure, exposure to chemical carcinogens such as tar and
pitch, as well as smoking, and a viral etiology, specifically the human papilloma virus, have all been proposed as
etiologic factors.
Q/Q(M)-476100 Report a Problem

Potrebbero piacerti anche